Firecracker Clinical Vignettes Flashcards

1
Q

A 45-year-old female with a history of atrial fibrillation develops sudden onset of left-sided hemiparesis and eye deviation to the right. She is brought to the emergency department where an evaluation with an MRI reveals a large area of restricted diffusion

A

The patient has suffered a middle cerebral territory artery stroke on the right (non-dominant) side. The left hemisphere is dominant (meaning that language localizes there) in almost all right-handed patients and 90% of left handed patients. Ischemic stroke in the MCA territory presents with basic exam findings, such as contralateral hemiplegia, eye deviation towards the side of the lesion (frontal eye fields), and contralateral neglect. Patients with large dominant hemisphere lesions can present with global aphasia, whereas non-dominant hemisphere infarcts result in anosogosia. Evaluation of ischemic stroke is performed with diffusion weighted MRI imaging. Anterior cerebral artery stroke commonly results in contralateral leg weakness. Posterior inferior cerebellar artery stroke can result in a Wallenberg (lateral medullary) syndrome.

How well did you know this?
1
Not at all
2
3
4
5
Perfectly
2
Q

A 44-year-old male presents to the hospital with a history of diffuse epigastric pain. He is found to have elevated serum amylase and lipase levels, in addition to abnormal liver function tests. An ultrasound is performed that demonstrates cholelithiasis without evidence for cholecystitis. He is admitted to the hospital for bowel rest and intravenous hydration. The following morning, all of his serum chemistries trend towards the normal range. A cholecystectomy is recommended, but the patient refuses. He feels better and is discharged to home. Two months later, he presents to the emergency department with right upper quadrant pain, fever to 104ºF, and jaundice. He is also slightly confused and is found to be hypotensive

A

The patient presents with symptoms of cholangitis. He has a history of gallstones as evidenced by his previous history of gallstone pancreatitis (self resolved). Cholangitis is identified by Charcot’s triad of right upper quadrant pain, fever, and jaundice. The patient also has the additional findings of mental status changes and shock (Reynold’s pentad) that is suggestive of sepsis. E. Coli and Klebsiella are the most common organisms. Cholecystitis is classically associated with prolonged right upper quadrant or epigastric pain, fever, abdominal guarding, a positive Murphy’s sign, and an elevated white blood cell count.

How well did you know this?
1
Not at all
2
3
4
5
Perfectly
3
Q

An 81-year-old woman is brought into the ED from her skilled nursing facility for the evaluation of fever. At baseline, she is nonverbal as the result of a left-sided CVA experienced many years ago. On arrival, her initial vitals are: T 39.05°C (102.3°F), HR 106, BP 132/74, RR 18, SpO2 98% on room air. A chest x-ray is obtained, which shows a dense consolidation in the right lower lobe. Given concern for a bacterial pneumonia, she is given 1 liter of normal saline and started on IV vancomycin, levofloxacin, and ceftriaxone. Approximately 25 minutes later, a nurse observes that the patient now has a new urticarial rash over her abdomen, trunk, and upper extremities. Repeat vitals are: T 39.1°C (102.4°F), HR 118, BP 104/76, RR 22, SpO291% on room air. On pulmonary exam she is found to have diffuse wheezing in all fields.

A. another liter of IV normal saline
B. IV diphenhydraine (Benadryl)
C. IV methylprenisoloe
D. IM epinephrnie
E. Supplemental oxygen via nasal cannula
F. Nebulized ipratropium and albuterol
A

This patient is experiencing an anaphylactic reaction, most likely to one of the antibiotics she received. Anaphylaxis is an acute, life-threatening, multisystem type 1 hypersensitivity reaction that most commonly presents with respiratory and skin findings. In severe reactions, patients can progress to distributive shock as a consequence of systemic vasodilation. This patient is demonstrating many of the key findings of anaphylaxis, including tachycardia, hypotension, rash, and bronchoconstriction. Of primary importance in anaphylaxis is ensuring a stable airway; were this patient to have findings on physical exam suggestive of angioedema or airway compromise, a low threshold should be maintained for emergent intubation.

While all of the answer choices listed are correct therapies in the treatment of anaphylaxis, the most important is intramuscular epinephrine, which will act quickly to reverse the life threatening bronchial and upper airway constriction

Wrong choices:
A. Another liter of IV normal saline
Patients in anaphylaxis may experience a rapid drop in their blood pressure as a consequence of histamine-induced vasodilation and fluid shifts, and may need several liters of IV fluid to maintain adequate perfusion pressures. However, as indicated above, IM epinephrine should come first.
B. IV diphenhydramine (Benadryl)
In anaphylaxis, mast cells throughout the body release histamine into the circulation. As such, antihistamines such as Benadryl play a key role in the appropriate management. Longer-acting second generation antihistamines, like cetirizine and loratidine, may also be employed for prolonged histamine blockade.
C. IV methylprenisolone
Systemic glucocorticoids, such as prednisone and methylprednisolone, are important to prevent the “rebound” anaphylactoid reaction that can occur several hours after the initial reaction as epinephrine and antihistamines begin to wear off.
E. Supplemental oxygen via nasal cannula
Inhaled bronchodilators are useful adjuncts for treating the symptomatic bronchoconstriction that can occur in anaphylaxis.
F. Nebulized ipratropium and albuterol
This patient is experiencing hypoxemia, likely as a result of both anaphylaxis and her pneumonia. Supplemental oxygen is warranted, but should not supersede the administration of epinephrine.

How well did you know this?
1
Not at all
2
3
4
5
Perfectly
4
Q

You are evaluating a six-month-old female infant at a routine well-child check. Her parents are concerned that she appears pale. She was born full-term via ceasarian section for breech presentation. She has no significant past medical history and her immunizations are up-to-date. She has reached all the appropriate developmental milestones. She was breast fed exclusive for the first two months, but since weaning she has only been fed organic goat’s milk. On physical exam, she is small for her age

A

The infant in this vignette has folate deficiency. She has a megaloblastic anemia and is getting an inadequate amount of folate in her diet. The main dietary sources of folic acid are animal products, leafy green vegetables, and fortified foods; however goat’s milk does not contain adequate amounts of folic acid.
B. Vitamin B12
Vitamin B12 deficiency causes a megaloblastic anemia similar to folate deficiency, but also presents with neurologic symptoms that in a young infant include developmental delay or regression, failure-to-thrive, weakness, seizures, and irritability.
C. Niacin (Vitamin B3)
Niacin deficiency causes pellagra, which is characterized by the three D’s: dermatitis, diarrhea, and dementia.

D. Riboflavin (Vitamin B2)
Riboflavin deficiency presents with cheilosis, glossitis, keratitis, photophobia, seborrheic dermatitis, sore throat, edema, hyperemia of mucous membranes, and a normocytic anemia.

E. Thiamine (Vitamin B1)
Thiamine deficiency causes Beriberi and Wernicke-Korsakoff syndrome. Beriberi presents with cardiac failure and/or neurologic manifestations including: irritability, peripheral neuritis, decreased tendon reflexes, loss of vibratory sense, aseptic meningitis, and ataxia. Wernicke encephalopathy presents with ophthalmoplegia, nystagmus, ataxia, intracranial hemorrhage, and confusion. Korsakoff psychosis results in short-term memory loss and confabulation with normal cognition.
F. Pyridoxine (Vitamin B6)
Pyridoxine deficiency causes refractory seizures, peripheral neuritis, dermatitis, and a microcytic anemia.

How well did you know this?
1
Not at all
2
3
4
5
Perfectly
5
Q

A 75-year-old man presents to the emergency department after having a bowel movement with bright red blood per rectum. He otherwise denies any abdominal pain, nausea, vomiting or hematemesis. He also denies recent weight loss, fevers, or night sweats, but he does report low energy for the past 2-3 days. On arrival to the emergency department, the patient’s temperature is 36.2°C (97.2°F), his heart rate is 122/minute, and his blood pressure is 92/50 mmHg. On physical exam, the patient appears pale but in no acute distress. His abdominal exam is unremarkable, and rectal exam demonstrates no visible external hemorrhoids but some bright red blood is noted in the rectal vault.

A. Mesenteric Ischemia
B. Peptic ulcer disease
C. Diverticulosis
D. Divertculitis
E. Colon Cancer
A

Diverticulosis technically refers only to the presence of colonic diverticula, although in common practice the term is used to describe the syndrome of painless diverticular bleeding. Diverticulosis is the most common cause of overt (rather than occult) lower GI bleeding in adults, and typically presents with painless, often large-volume, hematochezia. The majority of cases of bleeding are self-limited, although patients can rapidly develop symptomatic hypotension and anemia requiring transfusion. Given this patient’s age and likely large volume lower GI bleed (as suggested in this vignette by the combination of bright red blood pre rectum with hypotension, tachycardia, and pallor), diverticulosis is the most likely diagnosis. After initial resuscitation and stabilization, this patient should undergo colonoscopy to rule out colorectal cancer and confirm the diagnosis of diverticulosis. In addition, if bleeding does not stop spontaneously, endoscopic intervention can be attempted. In severe cases, endovascular or surgical intervention may be required.

A. Mesenteric ischemia
Mesenteric ischemia is a common cause of GI bleeding in the elderly that is caused by mesenteric vascular insufficiency, often secondary to atherosclerosis and thromboembolic disease. Mesenteric ischemia most commonly presents with severe abdominal pain, classically described as out of proportion to the findings on physical exam. This patient’s lack of pain makes mesenteric ischemia unlikely.
B. Peptic ulcer disease
Peptic ulcer disease most commonly presents with upper abdominal pain or discomfort, often in relation to eating (dyspepsia). Bleeding from a peptic ulcer (and most other upper GI sources) most often presents as melena (black tarry stool), rather than bright red blood. As this patient has neither pain nor melena, he is unlikely to have peptic ulcer disease.
D. Diverticulitis
Diverticulitis refers to inflammation of a colonic diverticulum. It presents with abdominal pain, usually localized to the left-lower quadrant, fever, nausea, vomiting, and sometimes hemodynamic instability and shock. On exam, the abdomen is tender to palpation, and peritoneal signs may be present. This patient is afebrile without any of the signs or symptoms of diverticulitis.
E. Colon Cancer
Colorectal cancer is the third most common type of cancer in both men and women in the United States. Symptoms may be insidious at first, and include bloody stools, weight loss, anemia, a change in bowel habits, and other systemic signs of inflammation. While colorectal cancer is the most common cause of lower GI bleeding in adults, most colorectal cancers present with occult bleeding. In contrast, diverticulosis is a more common cause of overt lower GI bleeding (bright red blood per rectum).

How well did you know this?
1
Not at all
2
3
4
5
Perfectly
6
Q

A 54-year-old male visits his primary care physician with the complaint of back pain. While he has had some pain in his back for months, he recently became more concerned when the pain seemed to travel down the back of his right thigh and into his calf. He complains of numbness in the right foot as well, mainly on the top of the foot and great toe. On exam he has normal ankle jerk reflexes and a weak extensor hallucis longus on the right.

  • L3/L4
  • L4/L5
  • L5/S1
  • Tethered spinal cord
  • Multi-level spinal stenosis
A

The patient suffers from a herniated lumbar disc at the L4/5 vertebral body level, corresponding to impingement of the L5 nerve root. The patient’s radicular symptoms in the L5 dermatome help to localize the pathology. The L5 dermatome typically affects the top of the foot with a unique motor distribution to the extensor hallucis longus. The L4 myotome affects dorsiflexion of the foot with the extensor hallucis longus not usually being affected. With S1 radiculopathy, patients often complain of sensory changes along the lateral aspect and bottom of their feet. The myotome distribution of the S1 nerve root focuses mainly on the gastrocnemius. Patients with an S1 radiculopathy will have weakness in their gastrocnemius (difficulty standing on their toes) and is accompanied by a diminished ankle jerk reflex. A normal MRI is unlikely given the prominent neurological signs. It would be unusual for a tethered spinal cord to present in adult life. Multi-level spinal stenosis results in neurogenic claudication.

How well did you know this?
1
Not at all
2
3
4
5
Perfectly
7
Q

A 15-month-old boy presents to his primary care physician after his parents observed during diaper changes that “his belly is sticking out more than usual.” They have also noticed “a black eye” despite no known trauma and “a bumpy rash.” The patient is growing and developing normally. His past medical history is only notable for constipation that has been treated with polyethylene glycol. His immunizations are up to date. His vital signs are temperature 36.8ºC (98.2ºF), pulse 102 beats/min, blood pressure 90/62 mm Hg, and respirations 22 breaths/min. Physical exam reveals left-sided proptosis and periorbital ecchymosis and diffuse, scattered, nontender subcutaneous nodules. The patient has a distended abdomen with hepatomegaly and a firm, irregular, non-tender abdominal mass that crosses the midline. There is no stool in the rectal vault. Cardiac and lung exam are unremarkable.

A. Urinary metanephrines and catecholamines
B. Urinary HVA and VMA
C. Serum alpha-fetoprotein (AFP)
D. Serum CBC
E. Serum beta-hCG
A

This patient has a firm, irregular abdominal mass that crosses the midline, as well as evidence of metastases to the orbital bones, liver, and skin. These findings, together with his age (50% of cases of neuroblastoma, with common sites including bone marrow, bone, liver, and skin.

Because neuroblastoma cells arise from neural crest cells, they often are able to take up and metabolize catecholamines, resulting in accumulation of VMA and HVA in the urine and serum. Evaluation for elevated urinary VMA and HVA serves as a sensitive and specific diagnostic test for neuroblastoma and is also useful for monitoring disease activity.
A. Urinary metanephrines and catecholamines
Urinary metanephrines and catecholamines (epinephrine, norepinephrine, and dopamine) are elevated with pheochromocytoma. Hypersecretion of catecholamines by this tumor, which arises from the adrenal medulla, results in episodic headaches, tachycardia, and sweating, typically accompanied by hypertension.
C. Serum alpha-fetoprotein (AFP)
Markedly elevated serum AFP levels are present with hepatoblastoma, the most common primary hepatic malignancy in children. The tumor most often occurs as a single liver mass, usually in the right lobe of the liver.
D. Serum complete blood count
Metastatic spread of neuroblastoma to bone marrow may result in blood count abnormalities, but this finding is not specific to neuroblastoma and thus does not establish the diagnosis.
E. Serum beta-human chorionic gonadotropin (beta-hCG)
Elevated serum beta-hCG levels occur with germ cell tumors. These neoplasms may be gonadal (arising from the ovary or testis) or extragonadal. Extragonadal germ cell tumors usually arise from midline sites, with sacrococcygeal and intracranial tumors being most common in children.

How well did you know this?
1
Not at all
2
3
4
5
Perfectly
8
Q

A 45-year-old man with a history of hypertension requiring three blood pressure lowering agents presents to his primary care physician complaining of “panic attacks.” The patient explains that the episodes have become more frequent and consist of sweating, headaches, and the sensation that his heart is racing. The symptoms make him feel anxious and he is wondering if he can get some medicine for anxiety. He has a blood pressure cuff at home and took his blood pressure during the last few episodes. His blood pressure was approximately 180/100 during the episodes and came down to his baseline of approximately 140/90 when he re-checked a couple hours after the episodes.

A

Von Hippel-Lindau, neurofibromatosis 1, MEN (multiple endocrine neoplasia) IIa, MEN IIb
This vignette describes the classic triad of symptoms seen in patients with a pheochromocytoma: episodic headache, sweating, and tachycardia. Other symptoms include palpitations, tremor, pallor, dyspnea, generalized weakness, and panic attack-type symptoms (particularly in pheochromocytomas that produce epinephrine).

Pheochromocytoma is associated with Von Hippel-Lindau, neurofibromatosis 1, MEN (multiple endocrine neoplasia) II and III (Note: MEN II is also referred to as MEN IIa; MEN III as IIb).

Pheochromocytoma is called a 10% tumor because 10% are malignant, 10% are found extra-adrenal, and 10% are bilateral. The other conditions mentioned do not have a known association with pheochromocytoma

How well did you know this?
1
Not at all
2
3
4
5
Perfectly
9
Q

You are evaluating a nine month-old female infant in a refuge camp who presents with irritability, poor appetite, and weakness. She was born via spontaneous vaginal delivery that was complicated by maternal hemorrhage and death. Her grandmother cares her for and feeds her tins of evaporated boiled milk. On physical exam, she is irritable and lying in a frog-leg position. Her weight and height are

A

The answer is Vitamin C deficiency also known as Scurvy. Boiling milk destroys the Vitamin C. Patient with Scurvy resents with irritability, developmental delay, decreased appetite, and fatigue. Patients then develop bleeding in the skin, mucous membranes, joints, muscle, or gastrointestinal tracts. Holding legs in the frog leg position may be caused by pain from subperiosteal hemorrhage, hemarthrosis, or muscle hemorrhage. Gingival disease begins as shiny, red, swollen gums that progress to necrosis and bleeding. Dermatologic findings include dystrophic hair, follicular keratosis, petechiae, purpura, ecchymoses, and poor wound healing. Children may also have hypertrophic cardiomyopathy, diminished adrenal and bone marrow function, and edema. Osteopenia is common in vitamin C deficiency and lateral metaphyseal spurs (corner sign) are pathognomonic. Other radiographic findings include: a white line of dense calcification at the metaphysis (Frankel lines), less dense transverse bands adjacent to Frankel lines (scurvy lines), thickened white line of calcification encircling the osteopenic epiphysis (Wimberger ring).

A. Vitamin A
Vitamin A deficiency causes night blindness, dry and scaly skin, and frequent infections.

C. Iron
Children typically have a mild microcytic anemic and iron deficiency from bleeding, however iron deficiency is not the primary problem.
D. Vitamin D
Vitamin D deficiency causes osteopenia, fracture, and rickets.

E. Vitamin E
Vitamin E deficiency causes neuropathy, retinopathy, and hemolytic anemia.
F. Vitamin K
Vitamin K deficiency causes bleeding from problems with blood clotting.

How well did you know this?
1
Not at all
2
3
4
5
Perfectly
10
Q

A 46-year-old male presents to his primary care physician with complaints of fatigue and skin itching. The symptoms have been gradually progressive over the course of a month. He has also lost about 10 pounds (unintentionally) and has some dull right upper quadrant pain in his abdomen. He has a history of ulcerative colitis.

  • primary sclerosing cholangitis
  • cholangitis
  • Mirizzi syndrome
  • primary biliary cirrhosis
  • cholangiocarcinoma
A

The patient suffers from primary sclerosing cholangitis. It is an autoimmune condition that usually affects men in their 4th and 5th decades of life. There is an association of this condition with ulcerative colitis. Patients present with symptoms related to impaired bile excretion and include fatigue, jaundice, pruritus, weight loss, and right upper quadrant pain. The diagnosis is made with ERCP, which shows multiple strictures and dilations of the biliary tree (as seen in this case).Eventually, this condition progresses to cirrhosis with the resultant development of portal hypertension.

Cholangitis presents with fever, right upper quadrant pain, and jaundice. Mirizzi syndrome is the compression of the common hepatic duct by a stone in the infundibulum of the gallbladder. Primary biliary cirrhosis is seen in women where the medium-sized hepatic ducts are affected. In contrast, in primary sclerosing cholangitis both intra and extra-hepatic ducts are affected. Cholangiocaricnoma typically occurs in elderly males, where painless jaundice is the most common presentation. A focal area of bile duct stenosis is usually seen.

How well did you know this?
1
Not at all
2
3
4
5
Perfectly
11
Q

A 34-year-old female presents to her obstetrician. She is 18 weeks pregnant. History is notable for 2 miscarriages as well as a deep vein thrombosis (DVT) at age 22 while she was on oral contraceptives. Her oral contraceptives were stopped and no other work-up was pursued. Two weeks later, the patient experiences heavy bleeding and the diagnosis of spontaneous abortion is made. Labs were notable for an elevated PTT.

A

Antiphospholipid antibody syndrome (also known as lupus anticoagulant syndrome) is a commonly tested acquired thrombophilia. In the blood stream, antibodies directly activate platelets and complement, leading to a hypercoagulable state. Patients commonly present with a history of recurrent DVT and miscarriages, such as in this vignette. In vitro, the antibodies interfere with phospholipids (hence antiphospholipid) and inhibit coagulation, leading to an elevated PTT. While a homozygote for factor V Leiden may experience this degree of clotting complications, it would be unlikely in a heterozygote. Protein C deficiency is rare and is not as commonly implicated in miscarriages. The other answer choices do not cause thrombophilia.

How well did you know this?
1
Not at all
2
3
4
5
Perfectly
12
Q

A 60-year-old woman with a 20-year history of celiac disease is evaluated for several months of progressive abdominal discomfort, bloating, nausea, subjective fever, chills, night sweats and a 10-pound weight loss. The patient was doing well with gluten restricted diet for a long period before the onset of these symptoms. On exam, she appears malnourished. Several enlarged lymph nodes are palpated in both the cervical and inguinal chains, and her abdomen is diffusely tender to palpation. Labs reveals hemoglobin of 10 g/dl with an MCV of 78 fl. Esophagogastroduodenoscopy and colonoscopy are performed, but inconclusive. CT scan shows luminal narrowing in the small bowel.

A. Crohn's Disease
B. Lymphoma of small bowel
C. Ulcerative Colitis
D. Small intestinal bacterial overgrowth
E. Short gut syndrome
A

Patients with a history of celiac disease are at increased risk of developing malignancy, specifically lymphoma of the small bowel. This patient with signs/symptoms of a small bowel obstructive process, in addition to constitutional symptoms, has a presentation concerning for intestinal lymphoma. Patients with celiac disease are also predisposed to developing other gastrointestinal cancers, such as esophageal squamous cell carcinoma, intestinal adenocarcinoma, colorectal carcinoma, and hepatocellular carcinoma.

A. Crohn’s disease
Crohn’s disease may involve the entire GI tract, and can cause strictures of the small bowel. Constitutional symptoms may also be seen. While this patient does have some of these features, her history of celiac disease predisposes her to development of lymphoma, which is the most important possibility to rule out.
C. Ulcerative colitis
Ulcerative colitis is characterized by bloody diarrhea, and may also cause constitutional symptoms. This patient lacks the bloody diarrhea, and has a history of celiac disease which is more closely associated with lymphoma, which provides a better explanation of her symptoms.
D. Small intestinal bacterial overgrowth
Small intestine bacterial overgrowth can result from a variety of conditions, including short bowel syndrome, chronic pancreatitis, immunodeficiency, liver disease, among other etiologies. Patients with small bowel bacterial overgrowth may develop vitamin B12 deficiency, which is associated with a macrocytic anemia. The constitutional symptoms experienced by this patient would not be expected with bacterial overgrowth.
E. Short gut syndrome
Short gut syndrome develops when there is a resection of a portion of the small bowel. This results in a malabsorptive state that is characterized by reduced transit time and diarrhea. There is no history of bowel surgery, and this patient’s symptoms cannot be entirely explained by nutrient deficiency.

How well did you know this?
1
Not at all
2
3
4
5
Perfectly
13
Q

A 55 year-old longshoreman with a past medical history of hypertension, hyperlipidemia, and coronary artery disease with two prior episodes of myocardial infarction presents to the emergency department with two hours of severe nausea, vomiting, and dizziness. He says that he was preparing for work this morning when he started choking on his coffee, and suddenly became acutely dizzy and nauseous. He attempted to walk to his bathroom to vomit, but lost his balance after taking only three steps and fell to the floor. He called out to his wife, who found him lying on the floor and called 911. In the ED, he continues to complain of severe nausea and room-spinning dizziness. Neurologic exam reveals hoarse, slurred speech, a left-sided ptosis, asymmetric but reactive pupils with the right pupil larger than the left, and a symmetric smile. Sensory exam shows preserved touch and vibratory sense throughout but absent pinprick and temperature sensation on the left face, right arm, and right leg. Strength is normal throughout the upper and lower extremities.

A. MCA
B. ACA
C. PCA
D. AICA
E. PICA
F. Basilar artery
G. ASA
A

Occlusions within the vessels of the posterior circulation can be some of the most challenging to diagnose and localize. The neuroanatomy of the brainstem and cerebellum is complex, and at times even experienced clinicians can get tripped up. The most important thing to do when presented with a posterior stroke is to recognize it as a possibility so that the appropriate diagnostic studies can be obtained in a timely manner. That being said, there are a few classic posterior stroke syndromes that are worth knowing.

The collection of symptoms described here is characteristic of the lateral medullary syndrome (Wallenberg syndrome), caused by the occlusion of the posterior inferior cerebellar artery (PICA). Keys to the diagnosis include the crossed sensory findings (ipsilateral facial deficits with contralateral body deficits), ipsilateral Horner’s syndrome, dysphagia, dysarthria, and cerebellar ataxia and vertigo.
A. Middle cerebral artery
Unlike the crossed deficits of subcortical stroke illustrated above, an occlusion of the middle cerebral artery will produce sensory and motor deficits of both the contralateral face and extremities.
B. Anterior cerebral artery
Anterior cerebral artery strokes most often cause contralateral lower extremity weakness and numbness, with varying degrees of behavioral and cognitive dysfunction.
C. Posterior cerebral artery
A posterior cerebral artery stroke typically causes major contralateral visual field deficits (often with macular sparing), as well as a number of higher cognitive visual deficits.
D. Anterior inferior cerebellar artery
An occlusion of the anterior inferior cerebellar artery (AICA) may produce some similar features to a posterior inferior cerebellar artery (ICA) stroke; however, gaze palsy, facial weakness, and an acute unilateral deafness are more likely to be present, while Horner’s syndrome, dysphagia, and dysarthria are unlikely to be found. Superior cerebellar artery occlusion may also overlap with the PICA and AICA occlusion syndromes, but is more likely to feature crossed deficits of touch and vibratory sense in addition to impaired pain and temperature sensation.
F. Basilar artery
Occlusion of the basilar artery, either from thrombosis or embolus, can block blood flow to the reticular activating system, resulting in an immediate and profound reduction in the level of consciousness. Depending on the precise location of the occlusion within the basilar artery, a variety of brainstem nuclei can be affected, producing a wide range of possible cranial nerve deficits. Hemiplegia and quadriplegia are common, as are a number of complex ophthalmoplegias. Lesions of the basis pontis that spare the tegmentum can produce a “locked-in” state, in which patients have preserved consciousness and cognition but only the most rudimentary motor control remaining (typically vertical eye movements and/or blinking). This is due to involvement of the pyramidal tracts.
G. Anterior spinal artery
Occlusion of the anterior spinal artery produces a “spinal cord stroke”, characterized by complete bilateral weakness below the level of the lesion. Pain and temperature sensation, but not fine touch or vibratory sensation, will also be lost below the level of the lesion. As the lesion occurs within the spinal cord, there are no cognitive, cranial nerve, or brainstem deficits.

How well did you know this?
1
Not at all
2
3
4
5
Perfectly
14
Q

A 2-year-old male presents with repeated episodes of burning, red, edematous skin shortly after sun exposure. No blisters are present. Urine porphyrins are normal, but total plasma porphyrin is elevated. The patient is treated with beta-carotene, and his family is encouraged to have him avoid sun exposure.

A. Acute Intermittent Porphyria
B. Porhpyria Cutanea Tarda
C. Hereditary Coproporphyria
D. Variegate Porphyria
E. Erythropoietic Protoporphyria
A

The porphyrias are disorders caused by reduced activity of enzymes in the heme biosynthetic pathway. They can be categorized by their dominant clinical presentations, which include photosensitivity symptoms (cutaneous porphyrias), neurovisceral symptoms (acute porphyrias), or a combination of photosensitivity and neurovisceral symptoms. Photosensitivity in cutaneous porphyrias is due to activation of porphyrins in the skin by visible and long wave ultraviolet light.

The acute, nonblistering photosensitivity seen in erythropoietic protoporphyria (EPP) is a distinctive feature from the other cutaneous porphyrias. While the diagnosis can be suspected when urine porphyrins are normal in the presence of an elevated total plasma porphyrin, these findings are not specific; the diagnosis is confirmed by demonstrating an elevated erythrocyte protoporphyrin level. Treatment involves sun avoidance and beta carotene administration.
A. Acute intermittent porphyria
Acute intermittent porphyria is an acute neurovisceral porphyria caused by mutations in the porphobilinogen deaminase (PBG-D) gene. Inheritance is autosomal dominant but with low penetrance, so family history does not always provide clues to the diagnosis. The presentation is variable with most patients never developing symptoms. The most common symptom is abdominal pain, sometimes with other gastrointestinal symptoms such as nausea, vomiting, and constipation. Other manifestations include peripheral neuropathies, mainly manifesting as weakness of the extremities, and autonomic nervous system involvement, with tachycardia as the most common sign.
B. Porphyria cutanea tarda
Porphyria cutanea tarda, the most common human porphyria, is a cutaneous porphyria resulting from deficient activity of hepatic uroporphyrinogen decarboxylase (UROD). It tends to present in mid- to late life with increased skin fragility and blistering lesions (bullae) on sun-exposed areas that leave scars and areas of hyper- and hypopigmentation. Hirsuitism is a common feature. In addition, most affected patients have mild elevations in hepatic transaminases, and advanced liver disease is seen in some older patients. Known exacerbating factors include iron, alcohol, smoking, estrogens, hepatitis C infection, and HIV infection.
C. Hereditary coproporphyria
Hereditary coproporphyria is due to mutations in coproporphyrinogen oxidase (CPO) and is the least common of the acute neurovisceral porphyrias. Acute neurovisceral attacks are indistinguishable to those in patients with acute intermittent porphyria but are generally milder. Some patients have cutaneous manifestations similar to those in porphyria cutanea tarda, but these are uncommon.
D. Variegate porphyria
Variegate porphyria results from subnormal activity of the mitochondrial enzyme protoporphyrinogen oxidase (PPO). It has both cutaneous and neurovisceral manifestations (the basis for the name “variegate”). It causes chronic, blistering skin lesions similar to those of porphyria cutanea tarda and neurovisceral symptoms identical to those of acute intermittent porphyria.

How well did you know this?
1
Not at all
2
3
4
5
Perfectly
15
Q

A 14-year-old girl presents to your clinic with a 3-week history of fever and pain in her wrists and knees. She has no significant past medical history, and her immunizations are up to date. On physical exam, her vitals signs are temperature 38.3°C (100.9°F), pulse 80 beats/min, blood pressure 140/86 mmHg, and respirations 16 breaths/min. She has an erythematous malar rash, periorbital edema, and several painless ulcers on the palate and tongue. She has warmth, tenderness, and moderate effusions of her wrists and knees bilaterally. Cardiac exam reveals a regular rhythm with a pericardial friction rub and no murmur or gallop. Her lungs are clear to auscultation bilaterally. Her abdomen is soft, nontender, and nondistended with no hepatosplenomegaly. A urinalysis reveals 3+ blood.

A. Rheumatoid Factor
B. Antinuclear antibody (ANA)
C. Anti-cyclic citrullinated protein (anti-CCP) antibody
D. Anti-double-stranded DNA (anti-dsDNA)
E. C3 and C4
F. Antineutrophil cytoplasmic antibodies (ANCA)

A

This patient meets 5 of 11 diagnostic criteria for SLE:

Malar rash
Oral ulcers
Serositis (pericarditis or pleuritis)
Arthritis
Nephropathy (as indicated by her hypertension, periorbital edema, and hematuria)

Other criteria include:

Discoid rash
Photosensitivity
Neurologic manifestations, such as seizure or psychosis
Hematologic manifestations, such as hemolytic anemia or lymphopenia
Presence of antinuclear antibody (ANA)
Immunologic manifestations, including anti-DNA, anti-Smith, or positive VDRL

Of the serologic tests performed to help diagnose SLE, anti-dsDNA is highly specific and is found in >75% of patients with SLE.

A. Rheumatoid factor (RF)
Rheumatoid factor (RF) is primarily evaluated in the diagnosis of rheumatoid arthritis; it is present in 75%-80% of patients with this disorder.
B. Antinuclear antibody (ANA)
Antinuclear antibody (ANA) is highly sensitive for systemic lupus erythematosus (SLE). It is positive in virtually all children with SLE. However, it is also found in patients with other rheumatologic diseases and up to one-third of healthy people. A higher ANA titer (≥1:1280) is more suggestive of SLE.
C. Anti-cyclic citrullinated protein (anti-CCP) antibody
Anti-CCP antibodies have a specificity of >90% and sensitivity of 50%-70% for rheumatoid arthritis.
E. C3 and C4 and CH50
Complement levels may be low in patients with SLE, but this finding is neither sensitive nor specific for the disease.
F. Antineutrophil cytoplasmic antibodies (ANCA)
Evaluation for ANCA has a central role in diagnosis of various vasculitides, including granulomatosis with polyangiitis (formerly known as Wegener’s granulomatosis).

How well did you know this?
1
Not at all
2
3
4
5
Perfectly
16
Q

An 18-year-old male is evaluated in the emergency department for dehydration. He recently went on a day hike, but was lost in the woods for 4 days with only one candy bar for food. His vital signs include a temperature of 37.3°C (99.14°F), heart rate of 100 beats/min, blood pressure 106/78 mmHg, respiratory rate 12 breaths/min and an oxygen saturation of 100% on room air. On exam, he is alert, awake, and oriented to person, time, and place. He appears thin and tired. His mucous membranes are dry. His heart is tachycardic with normal rhythm and his lungs are clear to auscultation bilaterally. His abdomen is soft, non-tender, and non-distended. His has generalized muscle weakness. He has badly sunburned skin over his neck, face and ears. During this period of starvation, this patient generated energy via the breakdown of protein from muscle tissue. Ammonia, a byproduct of amino acid catabolism, can be toxic at high levels. In spite of the increased levels ammonia produced during this period of starvation, this patient’s liver was able to convert it to urea for excretion in the urine.

One of the nitrogen groups found in urea is provided by ammonia and the other is provided by aspartate. What metabolic process is directly involved in providing a source of aspartate for the urea cycle?

A. glycolysis
B. citric acid cycle
C. beta oxidation
D. gluconeogenesis

A

Aspartate is an amino acid with multiple amino groups. It combines with citrulline to form argininosuccinate via the enzyme arginosuccinate synthetase. Argininosuccinate is subsequently cleaved to form arginine and fumarate. Arginine continues through the urea cycle, and fumarate enter the citric acid cycle. In the citric acid cycle, fumarate is converted to malate, which is then converted to oxaloacetate. Oxaloacetate re-combines with acetyl-CoA to form citrate and restarts the citric acid cycle. Oxaloacetate can also undergo transamination with glutamate, forming α-ketoglutarate and aspartate. Aspartate enters the mitochondrial matrix via the malate-aspartate shuttle and donates a second nitrogen group in the formation of urea.

A. Glycolysis
Glycolysis is the metabolic process that converts glucose to pyruvate; it does not provide a source of aspartate for the urea cycle.
C. Beta oxidation
Beta-oxidation is the metabolic process that breaks down fatty acid molecules to generate acetyle-coA, which is used by the citric acid cycle, and NADH and FADH2, which enter the electron transport chain; it does not provide a source of aspartate for the urea cycle.
D. Gluconeogenesis
Gluconeogenesis is the metabolic process that generates glucose from non-carbohydrate carbon substrates such as pyruvate, lactate, glycerol, and glucogenic amino acids; it does not provide a source of aspartate to the urea cycle.

How well did you know this?
1
Not at all
2
3
4
5
Perfectly
17
Q

A 64-year-old female is brought to her primary care physician by her husband who remarks, “There is something wrong with her head”. He says that she has problems remembering the names of various items around the house or how to conduct day-to-day tasks such as working the dishwasher. He also notices her occasionally wandering around the house in the middle of the night aimlessly, as if she were sleep walking. He says that her condition has been increasing in severity over time, and just this morning she seemed unable to recognize her niece who was visiting from out of town, precipitating the visit to the PCP. On examination, the patient is unable to correctly identify the year or current location, and she recalls zero of three unrelated objects at five minutes. She has a slightly blunted affect, but does not appear overly concerned about her condition. She incorrectly names a wristwatch a “clock”, and she cannot follow instructions to fold a piece of paper twice. Her husband comments that she has not had any major personality changes, nor has she had any urinary incontinence that he is aware of. The patient has a BMI of 32. Magnetic resonance imaging (MRI) of the head reveals a marked loss of cortical volume with a significant dilation of the lateral ventricles and subarachnoid spaces.

A. Normal Pressure Hydrocephalus
B. Non-communicating (obstructive) Hydrocephalus
C. Communicating hydrocephalus
D. Pseudotumor cerebri
E. Hydrocephalus Ex Vacuo
A

This patient’s history of progressive cognitive decline, aphasia, apraxia, and agnosia are highly suggestive of Alzheimer’s Disease, which is characterized microscopically by the accumulation of amyloid plaques and neurofibrillary tangles. This disease results in a progressive neurodegeneration with an accompanying atrophy of the cortex. The reduction in brain mass leaves an unoccupied space within the skull which is subsequently occupied by a relative increase in cerebrospinal fluid volume. This phenomenon is referred to as hydrocephalus ex vacuo, and does not occur exclusively in Alzheimer’s disease, but is rather part of any chronic neurodegenerative process that results in a significant loss in brain volume.

A. Normal pressure hydrocephalus (NPH)
Normal pressure hydrocephalus (NPH) is a particular form of communicating hydrocephalus, characterized by enlarged cerebral ventricles, with only intermittently elevated cerebrospinal fluid pressure. Patients with this disease tend to present with a frontal gait, urinary incontinence, and dementia (wet, wobbly, and wacky).
B. Non-communicating (obstructive) hydrocephalus
Obstructive hydrocephalus results from an obstruction within the ventricular system (hence the name), preventing the outflow of cerebrospinal fluid from its origin in the choroid plexus into the subarachnoid space. On imaging, obstructive hydrocephalus presents with large lateral ventricles with effacement of the sulci. There is usually a visibly obstructing lesion.
C. Communicating hydrocephalus
Communicating hydrocephalus is characterized by impaired cerebrospinal fluid reabsorption at the level of the arachnoid granulations in the absence of any flow obstruction within the ventricular system. On imaging, all the ventricles, including the cerebral aqueduct, are dilated.
D. Pseudotumor cerebri
Pseudotumor cerebri describes an idiopathic elevation in intracranial pressure in the absence of tumor or other disease. Imaging typically appears normal, although small or slit-like ventricles and an “empty sella sign” (flattening of the pituitary gland due to increased pressure) may be seen. It is classically seen in young women with obesity, and is diagnosed by lumbar puncture and assessment of the intracranial pressure. Papilledema is seen on fundoscopic exam.

How well did you know this?
1
Not at all
2
3
4
5
Perfectly
18
Q

A 55-year-old man with a history of cirrhosis undergoes a routine spinal surgery for degenerative intervertebral disc disease (lumbar discectomy). He has a history of liver cirrhosis with ascites secondary to hepatitis C, morbid obesity, as well as limited exercise capacity. Prior to surgery, he was evaluated with a cardiac stress test which was negative for ischemia. The procedure is technically uncomplicated and blood loss is estimated at

A

Prolonged Drug life
This patient suffers from a delayed emergence phenomenon from general anesthesia. This can cause much consternation amongst operating room personnel on both sides of the sterile drape, but it can be anticipated preoperatively in certain conditions. In patients with increased volume of distribution (defined as [ the total drug in the body / plasma concentration ]), drugs take on a longer half-life. Volume of distribution is commonly increased in cases of ascites and congestive heart failure that are associated with “third spacing” of fluid. In fact, one can track heart failure on a day-by-day basis by measuring a patient’s weight as an indicator of retained fluid. The other major variable that contributes to volume of distribution is the adipose composition of the patient. Drugs, such as propofol, distribute to adipose tissue when administered systemically. The half-life of a drug is estimated by the formula [ half-life = (0.7 * Volume of distribution)/ clearance ], therefore an increased volume of distribution will increase the half-life of a drug.

A. Propofol infusion syndrome
Propofol infusion syndrome is a condition mainly seen in pediatric patients. It can lead to cardiac failure, rhabdomyolysis, severe metabolic acidosis, and renal failure. For this reason, propofol is avoided in pediatric patients.
C. Higher drug clearance
Both a higher drug clearance and smaller volume of distribution would lead to a shorter half-life. When the half-life of an anesthetic agent is reduced, emergence from anesthesia is quicker, not delayed as seen in this patient.
D. Smaller volume of distribution
Both a higher drug clearance and smaller volume of distribution would lead to a shorter half-life. When the half-life of an anesthetic agent is reduced, emergence from anesthesia is quicker, not delayed as seen in this patient.
E. Centrilobular hepatic necrosis
Massive hepatic necrosis is a rare side effect of halothane, an inhaled anesthetic agent. It occurs 2 days to 3 weeks after anesthesia due to direct liver injury. Patients present with fever, anorexia, nausea, jaundice, and tender hepatomegaly. Lab values will reveal an elevated prothrombin time and elevated liver enzymes. This patient is presenting with delayed emergence from anesthesia; there is no evidence that suggests massive hepatic necrosis.
F. Cerebrovascular accident
Cerebrovascular accident is a rare complication during surgery and more commonly occurs during thoracic vascular cases secondary to embolic phenomena (e.g. aortic arch repair).

How well did you know this?
1
Not at all
2
3
4
5
Perfectly
19
Q

A 32-year-old female presents to the neurology clinic complaining of double vision. The patient reports that she has experienced intermittent tunnel vision and blurring of her left eye over the last three months, though she has no such symptoms now. Additionally, she has noticed a periodic tingling sensation in her neck, which he describes as “shock-like.” Physical examination reveals a horizontal gaze palsy in which the patient is unable to adduct her right eye when looking to the left. Abduction of the left eye is preserved, but accompanied by left-beating nystagmus. Fluid attenuated inversion recovery (FLAIR) magnetic resonance imaging reveals multiple white matter lesions of varying intensity. A lumbar puncture is performed to confirm the diagnosis.
What would be found in the CSF?
A. antibodies against aquaporin 4
B. Abnormal isotypes of myelin protein zero
C. Abnormal isotypes of myelin basic protein
D. Monoclonal B lymphocytes
E. Polyclonal immunoglobulins

A

E. Polyclonal immunoglobulins
This patient is suffering from internuclear ophthalmoplegia (INO), a classic presenting syndrome of multiple sclerosis (MS). Plaque formation in the white matter of the medial longitudinal fasciculus (MLF) prevents communication between the contralateral abducens nucleus (origin of CN VI) and the ipsilateral oculomotor nucleus (origin of CN III). A patient with a right-sided INO, when asked to look to her left, will be unable to adduct the right eye past midline, resulting in lateral conjugate gaze palsy and double vision. The left eye will have preserved abduction, however, a resulting unilateral nystagmus in the left eye is generally (but not always) observed. Conjugate gaze to the right will be preserved. In addition to INO, the patient complains of occasional blurring of vision and shocking sensations in the neck, indicative of the common MS-related conditions of transient optic neuritis and Lhermitte’s sign, respectively. This patient’s MRI also demonstrates the characteristic white matter lesions seen in MS.

MS is an autoimmune disease in which lymphocytes initiate a response against various white matter proteins. The reaction is generally immunoglobulin driven, and typically manifests itself in the form of a limited polyclonal expansion of IgG within the CSF; these oligoclonal immunoglobulins (detected as discrete bands on agarose gel electrophoresis) are present in approximately 95% of patients with MS.

A. Antibodies against aquaporin 4
Antibodies against aquaporin 4, the major aquaporin of astrocyte foot processes, are a characteristic of neuromyelitis optica. This rare autoimmune condition, which presents similarly to MS, is also known as Devic’s disease. Unlike in MS, lesions in this disease classically affect the spinal cord and optic nerves (as the name would suggest), with relative sparing of the cerebral cortices.
B. Abnormal isotypes of myelin protein zero
Abnormal isotypes or deactivating mutations of myelin protein zero may be seen in some forms of Charcot-Marie-Tooth disease or Dejerine-Sottas disease. This protein normally interacts with peripheral myelin protein 22 (PMP22) to provide structural integrity to Schwann cell-generated peripheral myelin, and would not likely be seen in cerebrospinal fluid of a patient with MS.
C. Abnormal isotypes of myelin basic protein
Antibodies against myelin basic protein (MBP) may be seen in multiple sclerosis, but are not pathognomonic. Mutations and abnormal isotypes of MBP are not associated with any major disease.
D. Monoclonal B lymphocytes
As MS is an antibody driven autoimmune disease, B lymphocyte expansion is expected. However, the overactive B cells in MS have a polyclonal origin, each producing an antibody targeted against a different epitope of the offending antigen. Monoclonal B cell expansion indicates a common progenitor cell, and suggests an alternative diagnosis. Such a population may be seen in the context of central nervous system B cell lymphoma, often a consequence of prolonged immunosuppression of deficiency, as in patients with AIDS.

How well did you know this?
1
Not at all
2
3
4
5
Perfectly
20
Q

A 22-year-old female presents to her primary care physician with “episodes” that consist of severe intermittent headaches, diaphoresis, and palpitations. While in the office, she has one of these episodes. Her systolic blood pressure during the episode is measured to be 210 mmHg. A CT scan reveals a lesion in her right adrenal gland.

This patient’s symptoms are due to excess secretion of biogenic amines from malignant cells. Which enzyme catalyzes the rate-limiting step in the synthesis of these biogenic amines?

A. PMNT
B. MAO
C. Tyrosine Hydroxylase
D. DOPA decarboxylase
E. dopamine beta hydroxylase
A

C. Tyrosine hydroxylase
The rate-limiting step in catecholamine synthesis is catalyzed by tyrosine hydroxylase, which mediates the conversion of tyrosine to DOPA (dihydroxyphenylalanine).

How well did you know this?
1
Not at all
2
3
4
5
Perfectly
21
Q

A 49-year-old woman presents with right leg pain that is worse when bearing weight. She has a past medical history notable for progressive chronic kidney disease, but has not been under the care of a physician for many years. The patient shares that she went through menopause one year ago and has elected to forgo estrogen replacement therapy. Laboratory tests indicate low levels of 25-OH cholecalciferol, calcium, and phosphorus, and an elevated alkaline phosphatase and PTH.

An x-ray demonstrates bone demineralization, but no evidence of acute fracture. What is the most likely diagnosis?

A. Osteoporosis
B. Osteomalacia
C. Osteopetrosis
D. Paget's disease
E. Primary Hyperparathyroidism
A

B. Osteomalacia
Vitamin D deficiency results in osteomalacia in adults, and rickets in children. These are characterized by defective mineralization/calcification of osteoid leading to soft bones.
A deficiency is evidenced in this patient by the low level of 25-OH vitamin D, the storage form of the vitamin. Vitamin D3 (cholecalciferol) is predominantly derived through the consumption of milk, and is also formed in the skin with sun-exposure, with subsequent hydroxylations in the liver and kidney. The active form of vitamin D is 1,25-(OH)2 D3 or calcitriol.
In this country osteomalacia is an unusual disease, as patients with chronic kidney disease are closely monitored for their calcium and phosphorus levels. However, it may be seen in a patient with worsening renal failure who is not being followed by a physician.
Active vitamin D functions in the body to increase the intestinal absorption of calcium and phosphate. Active vitamin D induces calbindin D-28K (Ca2+ binding protein). Vitamin D also increases bone resorption, providing calcium and phosphate from ‘old’ bone to mineralize ‘new’ bone. Finally, active vitamin D increases renal reabsorption of Ca2+ and phosphate (minor effect). In children, rickets causes bone pain and deformity. In adults, osteomalacia presents as bone pain and muscle weakness. Osteoblasts are hyperactive, resulting in increased alkaline phosphatase.
Osteomalacia and rickets are both treated with vitamin D supplementation.

A. Osteoporosis
Osteoporosis is a loss of trabecular (spongy) bone despite normal bone mineralization and lab values (serum Ca2+ and PO43-). Type 1 osteoporosis occurs in postmenopausal women, when a decrease in estrogen levels increases net bone resorption by increasing osteoclast activity and decreasing osteoblast activity. Bone will lose mass and interconnections, which can lead to vertebral crush fractures. It may present as acute back pain, a loss of height, and kyphosis.

C. Osteopetrosis, also known as marble bone disease, is the failure of normal bone resorption as the result of defective osteoclasts. A mutation of carbonic anhydrase II impairs the ability of osteoclasts to generate the acidic environment necessary for bone resorption. Bone is thickened, dense, and prone to fracture. An x-ray shows a ‘bone-in-bone’ appearance. Unlike the above patient, osteopetrosis will produce normal phosphate and PTH levels (this patient has decreased serum phosphate and increased

D. Paget’s disease of bone is a localized disorder of bone remodeling. It is caused by an increase in both osteoblastic and osteoclastic activity. Like the above patient, alkaline phosphatase is elevated; however, serum calcium, phosphorus and PTH levels are normal. Bone will appear with a mosaic “woven” pattern.
E. Primary Hyperparathyroidism.
The elevated PTH in this patient is secondary to the depressed levels of serum Ca2+, and is not the primary disorder. Primary hyperparathyroidism would instead lead to elevated levels of active vitamin D, because PTH stimulates 1α-hydroxylase in the kidney. Other laboratory findings would include high serum Ca2+ and decreased serum phosphate (PTH leads to increased calcium, but decreased phosphate reabsorption in the kidney).

How well did you know this?
1
Not at all
2
3
4
5
Perfectly
22
Q

A 43-year-old male presents to the clinic with a maculopapular rash that started 2 days ago. The rash is non-pruritic, appearing on his forearms, wrists, and ankles. He reports that he is also experiencing fever, headache, myalgia, nausea and vomiting. He states that he went on a camping trip last week, but he was asymptomatic and did not notice any rash during the trip. Labs revealed mild leukocytosis, thrombocytopenia, mildly elevated aminotransferase levels, and elevated BUN. Lumbar puncture did not reveal the presence of a pathogen, but did show an elevated WBC count (100 cell/uL; predominantly lymphocytes) and elevated protein (150 mg/dL).

What is the next best step in the management of this patient?

A. Send a punch biopsy for indirect immunofluorescence assay (IFA)

B. Prescribe doxycycline

C. Send a blood sample for PCR

D. Prescribe ceftriaxone

A

This patient’s acute rash, systemic symptoms, and recent history of outdoor activity are highly indicative of Rocky Mountain Spotted Fever (RMSF), a tick-borne disease caused by the bacterium Rickettsia rickettsii. Symptoms first appear typically 2-14 days after the bite of an infected tick, but many patients do not know that they have been bitten. Therefore, obtaining a detailed history is very important. RMSF is a serious illness that can be fatal in the first 8 days of symptoms if not treated accordingly, so if the disease is suspected clinically, doxycycline should be the immediate first line of treatment. A punch biopsy should then be sent for IFA to confirm the presence of the R. rickettsii antigen, which would provide a definitive diagnosis. However, in many cases an early titer will be low or negative, which is why treatment should begin while a second titer is obtained 2-4 weeks later.

A. Send a punch biopsy for indirect immunofluorescence assay (IFA)
A punch biopsy for IFA should be performed, but that alone is not sufficient. Since RMSF is such a rapid and serious disease, treatment with doxycycline should begin even before laboratory confirmation.

C. Send a blood sample for PCR
R. rickettsii infects endothelial cells lining blood vessels, but they do not circulate in large numbers in the plasma unless the patient has progressed to a very serious stage of infection. Since the patient presented very early in the clinical course, blood specimens will often show negative results for PCR or culture. If a rash is present, a skin biopsy is always preferred to blood sample for serologic testing.

D. Prescribe ceftriaxone
Use of antibiotics other than doxycycline is associated with a higher risk of fatality. Therefore, doxycycline should always be the first-line of treatment for adults and children alike as soon as RMSF is suspected.

How well did you know this?
1
Not at all
2
3
4
5
Perfectly
23
Q

A 25-year-old man develops recurrent episodes of unilateral, retro-orbital pain. He describes the headache as “the worst pain imaginable.” He gets these headaches every few days for a period of weeks, but can then go several months without experiencing one. During the attacks, he develops ipsilateral rhinorrhea, lacrimation, and facial sweating.
What treatment can be used to abort one of these attacks?

A. Carbamazepine
B. Oxygen therapy
C. Ibuprofen
D. Acetaminophen
E. Caffeine
A

B. Oxygen therapy
This patient is most likely suffering from cluster headaches. Cluster headache is a relatively uncommon recurrent headache syndrome. Unlike migraine headaches, cluster headaches occur more commonly in men. Cluster headaches are generally unilateral, and are accompanied by autonomic dysfunction such as lacrimation, rhinorrhea, and flushing/sweating. High-flow oxygen is a commonly employed abortive therapy for cluster headache.

A. Carbamazepine
Carbamazepine is an anti-epileptic drug that is commonly used for the treatment of trigeminal neuralgia. Unlike cluster headaches, trigeminal neuralgia is characterized by brief episodes of sharp, electric-like unilateral facial pain lasting seconds to minutes. Pain is maximal at the time of onset, and is triggered by innocuous stimuli, such as touching the face. This patient’s symptoms are more consistent with a diagnosis of cluster headache than trigeminal neuralgia.

C. Ibuprofen
Ibuprofen is a non-steroidal anti-inflammatory drug (NSAID), useful for the treatment of many common aches and pains. Though it may provide minor relief for cluster headaches and other recurrent headache syndromes, it is unlikely to be as effective as high-flow oxygen for the treatment of cluster headache.
D. Acetaminophen
Acetaminophen is the active ingredient in Tylenol. It is frequently used, along with oral NSAIDs, as an abortive treatment for mild to moderate headaches. It is unlikely to be as effective as high-flow oxygen for the treatment of cluster headache.

E. Caffeine
Caffeine is a stimulant found in many common foods and drinks (coffee, chocolate, tea). It is frequently used in combination with acetaminophen and an NSAID for the treatment of mild to moderate headache pain. It is unlikely to be as effective as high-flow oxygen for the treatment of cluster headache.

24
Q

A 14-year-old girl presents as a new patient for a well-child checkup. Her height is in the 5th percentile for her age. Her vital signs are temperature 36.9ºC (98.4ºF), pulse 70 beats/min, blood pressure 114/64 mm Hg, and respirations 13 breaths/min. On physical exam, her neck is short with a webbed appearance.Her heart sounds are regular with a systolic ejection murmur that is best heard at the right upper sternal border. Her chest appears broad with widely spaced nipples. Lungs are clear to auscultation. Abdomen is soft and non-distended with no hepatosplenomegaly.She has Tanner stage I breast development and Tanner stage 4 pubic hair development. She is developmentally appropriate with no abnormalities on neurological exam. Skin exam shows multiple pigmented nevi.
Which of the following cardiac anomalies is associated with this syndrome?

A. Atrioventricular septal defect
B. Truncus arteriosus
C. Pulm artery stenosis
D. Peripheral pulm stenosis
E. Aortic stenosis
A

Turner syndrome is caused by partial or complete monosomy of the X chromosome with either a 45,X karyotype or mosaicism. Cardiac anomalies in Turner syndrome include aortic stenosis, bicuspid aortic valve, coarctation of the aorta, and mitral valve prolapse.

Turner syndrome is associated with a wide range of other features, including short stature, a webbed neck and low-set posterior hairline, a broad chest (“shield chest”) with widely spaced nipples, and multiple pigmented nevi. The gonads are usually streaks of fibrous tissue (a condition called gonadal dysgenesis). As a result, affected patients generally have primary amenorrhea and lack of breast development. Adrenarche with resulting pubic hair development occurs at a normal age.

A. Atrioventricular septal defect
Atrioventricular septal defects are not associated with Turner syndrome. They are common in patients with Down syndrome (trisomy 21). About half of patients with Down syndrome have congenital heart defects, which may also include ventricular septal defects, tetralogy of Fallot, and patent ductus arteriosus.
B. Truncus arteriosus
Truncus arteriosus is not a cardiac anomaly associated with Turner syndrome. It is one of the conotruncal defects associated with a deletion in chromosome 22q11, which results in DiGeorge or velocardiofacial syndrome. Other possible conotruncal defects include an interrupted aortic arch, tetralogy of Fallot, and double-outlet right ventricle.
C. Pulmonary artery stenosis
Pulmonary artery stenosis is not a cardiac anomaly associated with Turner syndrome. It is one of the defects associated with Noonan syndrome, along with dysplastic pulmonary valve stenosis and atrial septal defects.
D. Peripheral pulmonary stenosis
Peripheral pulmonary stenosis is not a cardiac anomaly associated with Turner syndrome. It is one of the defects associated with Williams syndrome, along with supravalvular aortic stenosis.

25
Q

A 21-year-old single African-American male is referred to a psychiatrist by his primary care doctor for concerns of psychotic illness. When he presents to the appointment, he is appropriate, his history answers are linear and organized, and he denies hallucinations. He reports that he is working full-time at a video store. However, his father, who accompanies him, states that about one month ago, there was a two-week period during which the patient “acted very strange”. His father goes on to describe that during that time, the patient became convinced that the mayor and the police chief were conspiring against him to prevent him from finding work. In addition, he was observed by his family to be “carrying on whole conversations when no one else was in the room”.The father also reports that, during this time, the patient’s speech was “garbled, like he was jumping all around in his head and we couldn’t follow what he was saying”. At the conclusion of the interview, the patient’s father says, “Honestly doc, I don’t even know why we’re here. My son is back to normal.” The patient denies present or previous substance use, and reports that his mood is “good”, and did not change during his symptomatic period.
DSM-V diagnosis?

A. Schizophrenia
B. Schizoaffective disorder
C. Schizophreniform disorder
D. Brief psychotic disorder
E. Substance/medication-induced psychotic disorder
A

D. Brief psychotic disorder
Brief psychotic disorder involves the sudden onset of one or more of the positive psychotic symptoms such as delusions, hallucinations, disorganized speech or disorganized behavior. The duration of the illness is greater than one day, but less than one month, and the patient returns to the pre-morbid level of functioning.
Patients must meet all three of the following DSM-5 criteria for brief psychotic disorder.
A
Presence of one (or more) of the following: 1) delusions, 2) hallucinations, 3) disorganized speech, 4) disorganized or catatonic behavior. *Patients must have either 1,2, or 3 to meet the criteria for the diagnosis.
B
Duration of symptoms > 1 day but

26
Q
A 55-year-old female with type 2 diabetes takes a medication that acts by decreasing the rate of hepatic gluconeogenesis. Shortly after starting this medication, she experienced severe episodes of diarrhea. She was ultimately transitioned to the extended-release formulation of the drug.
A. Sulfonylureas
B. Thiazolidinediones
C. Alpha glucosidase inhibitors
D. Biguanides
E. Meglitinides
A

This patient is most likely taking metformin, one of the most commonly prescribed medications for type 2 diabetes. Metformin is a biguanide, a class of medications that act by decreasing hepatic gluconeogenesis (and to a lesser extent by increasing the insulin-sensitivity of muscle, adipose, and hepatic tissue). Biguanides primarily act by activating the AMP-activated protein kinase (AMPK), which ultimately suppresses hepatic gluconeogenesis. Because they decrease hepatic gluconeogenesis and increase insulin sensitivity without directly increasing insulin secretion, biguanides are less likely to cause hypoglycemia than sulfonylureas or exogenous insulin. Side effects of biguanides include diarrhea and lactic acidosis. Though rare, lactic acidosis associated with biguanide use is a serious and potentially life-threatening complication, and their use is cautioned in patients with impaired renal function and heart failure.

A. Sulfonylureas

              Sulfonylureas, such as glyburide and glipizide, function by inhibiting the ATP-gated potassium channel located in the pancreatic beta cell membrane. When the potassium channel closes, the cell becomes depolarized, causing voltage-gated calcium channels to open. The influx of calcium ultimately triggers the release of insulin. Because sulfonylureas increase serum levels of insulin, they can cause hypoglycemia. Sulfonylureas do not typically cause diarrhea.

              B. Thiazolidinediones

              The thiazolidinediones, rosiglitazone and pioglitazone, increase the insulin-sensitivity of muscle, adipose, and hepatic tissue by binding to the peroxisome proliferator-activated receptors (PPARs). Their main side effects are weight gain and fluid retention, which can exacerbate congestive heart failure in some patients. Additional side effects include an increased risk of fractures and hepatotoxicity. Thiazolidinediones do not typically cause diarrhea.

              C. Alpha-glucosidase inhibitors

              The alpha-glucosidase inhibitors, acarbose and miglitol, inhibit the action of alpha-glucosidase in the GI tract to impair the breakdown of complex polysaccharides into monosaccharides. This slows the postprandial rise in blood glucose level. Though diarrhea is one of the main side effects seen with this class of medications, their mechanism of action is distinct from the drug described in this vignette. 

              E. Meglitinides

              The meglitinides, repaglinide and nateglinide, act similarly to the sulfonylureas by binding to and inhibiting the activity of the ATP-gated potassium channel to increase the secretion of insulin from the pancreatic beta cells. They do not typically cause diarrhea.
27
Q

A 16-year-old girl with a past medical history of depression presents in the emergency department after a suicide attempt. According to Emergency Medical Services, she was found with a half-empty bottle of pills that she had purchased a few hours ago at the local pharmacy. Her temperature is 38.5°C (101.3°F), heart rate is 140 beats per minute, and respiratory rate is 30 beats per minute. She appears confused, her pupils are dilated, bowel sounds are decreased, and her skin is flushed and dry.
Which of the following medications is she most likely to have ingested?
A. Vit C
B. Aspirin
C. Diphenhydramine
D. Acetaminophen
E. Caffeine pills

A

C. Diphenhydramine
This patient has most likely ingested diphenhydramine, an over the counter medication with anticholinergic effects. The mnemonic: “hot as a hare, mad as a hatter, red as a beet, dry as a bone, blind as a bat, and full as a tick” is helpful for remembering the toxidrome associated with ingestion of cholinergics.

A. Vitamin C

              Immediate effects of Vitamin C overdose are nausea and diarrhea.

              B. Aspirin

              Aspirin toxicity causes fever, diaphoresis, tachycardia, GI upset, confusion and agitation with severe toxicity causing pulmonary edema, coma, and seizures.

              D. Acetaminophen

              In the first 24 hours, acetaminophen overdose presents with nausea, vomiting, diaphoresis and malaise, but an otherwise normal physical exam.

              E. Caffeine pills

              Caffeine overdose typically presents with tachycardia, agitation, mydriasis, palpitations, and increased bowel sounds.
28
Q

A 28-month-old boy presents to the emergency department with a 1-day history of fever and worsening shortness of breath. His parents say that he seemed healthy during his first year of life, but he then began to have recurrent pneumonias, resulting in 3 hospitalizations since 12 months of age, and recurrent otitis media. His vital signs are temperature 39.5ºC (103.1ºF), pulse 140 beats/min, blood pressure 90/62 mm Hg, respirations 34 breaths/min, and oxygen saturation 93% on room air. His weight is below the 3rd percentile. He is ill but non-toxic-appearing and using accessory muscles to breathe. His oropharynx is nonerythematous, and tonsils are absent. He has a normal S1 and S2 with no murmur, rub, or gallop. Lung exam reveals decreased breath sounds over the right lower lung field. A chest x-ray shows a right lower lobe consolidation. He is admitted to the hospital for further evaluation and treatment.
Which of the following is most likely to be seen on laboratory evaluation of peripheral blood?
A. Deficient neutrophil function
B. Low or absent B cells and immunoglobulins
C. Low or absent T cells
D. Low or absent natural killer (NK) cells
E. Low immunoglobulins G, A, and E and normal or high immunoglobulin M

A

B. Low or absent B cells and immunoglobulins
This patient’s history of recurrent sinopulmonary infections, absent tonsils, and poor growth are consistent with Bruton’s X-linked agammaglobulinemia. The condition is caused by mutations in the gene for Bruton tyrosine kinase, which is important for B-cell development and expansion. The characteristic laboratory findings are low or absent B cells in peripheral blood and lymphoid tissues and very low or undetectable levels of serum immunoglobulins. Patients with X-linked agammaglobulinemia are prone to acute infections with encapsulated bacteria (e.g., Streptococcus pneumoniae, Haemophilus influenzae, Streptococcus pyogenes) and to chronic enterovirus infections. Infections typically start later in infancy after waning of maternally-derived antibody.
A. Deficient neutrophil function
Deficient neutrophil function—specifically, decreased neutrophil superoxide production—is found in chronic granulomatous disease, a condition caused by genetic defects in phagocyte nicotinamide adenine dinucleotide phosphate (NADPH) oxidase. Affected patients are prone to granuloma formation and recurrent bacterial and fungal infections with catalase-positive organisms, most commonly Staphylococcus aureus, Burkholderia cepacia complex, Serratia marcescens, Aspergillus species, and Nocardia species.
C. Low or absent T cells
Very low or absent T cells are a feature of severe combined immunodeficiency (SCID), a syndrome caused by defects in genes the products of which are necessary for function of both B and T cells. Patients present in the first year of life with persistent mucocutaneous candidiasis and often life-threatening infections with opportunistic pathogens (e.g., Pneumocystis jirovecii) and common viruses (e.g., adenovirus, respiratory syncytial virus, herpesviruses, norovirus, rotavirus), as well as chronic diarrhea and failure to thrive.

D. Low or absent natural killer (NK) cells
Low or absent NK cells are found in NK cell deficiency syndromes. Isolated NK cell deficiencies, which are rare, may be due to low numbers of NK cells or normal numbers of NK cells with defective function. Affected patients experience severe or recurrent infections with herpesviruses (cytomegalovirus, Epstein Barr virus, herpes simplex viruses, and varicella zoster virus).
E. Low immunoglobulins G, A, and E and normal or high immunoglobulin M
Low immunoglobulins G, A, and E and normal or high immunoglobulin M are found in the hyperimmunoglobulin M syndromes, a group of rare, heterogeneous conditions caused by defects in immunoglobulin class-switching. Affected patients have poor antibody function and recurrent infections starting in infancy, including sinopulmonary infections and infections with opportunistic pathogens, especially Pneumocystis jirovecii, Cryptosporidium, and Histoplasma.

29
Q

A 4-year-old girl presents to the emergency department with excruciating pain in her right leg for the past 12 hours. Her past medical history is notable for Sickle Cell Disease (HbSS). On physical examination, her temperature is 38.5°C (101.3°F), heart rate is 111 beats per minute, blood pressure is 110/78 mmHg, respiratory rate is 24 breaths per minute, and oxygen saturation is 99% on room air. There is erythema and moderate swelling over the upper medial surface of the right tibia with point tenderness. Laboratory evaluations are:

		WBC
		16,000 cells/mL

		hemoglobin
		12 mg/dL

		hematocrit
		36%

		platelets
		235,000 cells/mL

		C-reactive protein
		96 mg/L

Plain films of the tibia are normal.

A. Salmonella Osteomyelitis
B. Staph Aureus Osteomyelitis
C. Avascular necrosis
D. Acute pain crisis
E. Fracture of the tibia
A

A. Salmonella osteomyelitis
This child’s presentation is very concerning for osteomyelitis as she has fever, erythema, swelling, and point tenderness over the right tibia. She also has an elevated white blood cell count and C-reactive protein. Early osteomyelitis can often not be diagnosed with a radiograph. Patients with sickle cell are at increased risk for salmonella infections due to their functional asplenia.

B. Staph Aureus osteomyelitis
Staph Aureus osteomyelitis is the most common cause of all cases of osteomyelitis. In sickle cell disease patients, however, it only accounts for 25% of cases of osteomyelitis.

C. Avascular necrosis
The plain films are normal making AVN unlikely. Also, this is an unusual location for AVN.

D. Acute pain crisis
​The erythema and swelling make this less likely to be an acute pain crisis.

E. Fracture of the tibia
The plain films are normal making fracture unlikely. Also, there is no history of trauma to her leg.

30
Q
A 7 day old child born to a G2P1-->2 mother via scheduled Caesarian section is seen by her pediatrician for a well child check. She has been feeding well and gaining weight appropriately since discharge from hospital. On physical exam, the pediatrician notes a systolic ejection murmur over the apex of the heart and weak femoral pulses. A blood pressure taken in the right upper extremity is 144/91 mmHg. This infant should be screened for which of the following genetic disorders?
A. 22q11.2 deletion
B. XXY
C. Trisomy 21
D. 45, XO
A

D. 45, XO
Genetic defect of Turner syndrome. Infant here has clinical signs of a symptomatic aortic stenosis including classic heart murmur, weak femoral pulses, and upper extremity hypertension. Congenital heart disease- coarctation of the aorta, aortic stenosis, bicuspid aortic valve. Short stature, webbed neck, low posterior hairline, misshapen or rotated ears, a narrow palate with crowded teeth, a broad chest with widely spaced nipples, cubitus vagus, hyper convex nails, multi pigmented nevi, and pubertal delay
A. 22q11.2 deletion- DiGeorge syndrome. Associated cardiac defects include tetralogy of Fallot, interrupted aortic arch, but not aortic coarctation
B. XXY- Klinefelter’s syndrome, which does not have associated cardiac defects
C. Trisomy 21 or Down syndrome, associated with endocardial cushion defects and septal defects

31
Q

A 35-year-old man with a known history of HIV presents to the urgent care clinic for one day of fever, cough, and chest pain. He is currently on HAART therapy, and has not required any medical care in the past year. His most recent CD4 count was 400/μl. He denies alcohol, tobacco, and illicit drug use. Vital signs include: temperature 38.6°C (101.48°F), blood pressure 124/78 mmHg, pulse 105 beats/minute and respiratory rate 18 breaths per minute. On physical exam, he is ill-appearing and has crackles in his right lung base. A chest radiograph shows consolidation in the right lower lobe.

What organism is most likely responsible for his illness?

A. Pneumocystis jirovecii
B. Mycobacterium tuberculosis
C. Streptococcus pneumoniae
D. Klebsiella pneumoniae
E. Mycoplasma pneumoniae
F. Mycobacterium avium complex
G. Coccidiomyocosis
A

The patient in this vignette shows signs and symptoms of a community acquired pneumonia. This patient’s HIV has been well-controlled on antiretroviral therapy and he has not had any hospitalizations that would expose him to resistant organisms. S. pneumoniae remains the most common bacterial origin of invasive respiratory in both adults and children with HIV. In addition to opportunistic infections, patients with HIV are still susceptible to common pathogens.

A. Pneumocystis jirovecii
Pneumocystis jirovecii is the most common opportunistic infection causing pneumonia in HIV patients, but is more likely in patients with a CD4 cell count

32
Q

A 55-year-old female factory worker presents to the emergency department complaining of severe left upper arm pain that began earlier that morning. Three days ago she sustained a deep penetrating wound to her upper arm from a broken machine at her workplace. At the time, she bandaged the wound and has been changing the bandage daily She did not seek medical attention. Her physical examination is significant for pain, warmth, induration, and crepitus of the left upper arm. There is no cervical or axillary lymphadenopathy. A radiograph of her left arm reveals gas in the soft tissue of the upper arm. She has a white blood cell count of 22,000/ul with 90% neutrophils and a creatine kinase of 1,434 IU/L with a normal heart muscle fraction. An electrocardiogram is normal. Intravenous penicillin and clindamycin are started, and she is taken to the operating room, where necrotic muscle tissue is found in the left arm.
The toxin produced by the infectious agent implicated in this patient works by which of the following mechanism?

A. Inhibits protein synthesis
B. Disrupts cytoskeleton by depolymerizing actin filaments
C. Increases cAMP
D. Acts as a protease

A

E. Disrupts the cell membrane
This woman has traumatic clostridial myonecrosis, commonly referred to as gas gangrene. It is caused by Clostridium perfringens, a non-motile anaerobic gram-positive bacilli, which can also cause food poisoning. Trauma allows clostridial spores to enter directly into deep tissue. The mean incubation period is less than 24 hours. Once optimal growth conditions are reached, the organism elaborates an alpha toxin, which cleaves lecithin, a key component of cell membranes. An elevated creatine kinase level, as seen in this case, indicates the presence of muscle damage. The findings of rapid progression of the cellulitis, severe pain at the site of infection, presence of crepitus, radiographic observation of gas in the tissue, and finding of necrotic tissue at surgery are all consistent with clostridial myonecrosis.

A. Inhibits protein synthesis

              Toxins that inhibit protein synthesis include

Exotoxin A of Pseudomonas spp. that acts by ADP ribosylation of EF2. Pseudomonas spp. cause a variety of different types of infections including: pneumonia in patients with chronic lung disease (e.g. cystic fibrosis patients), urinary tract infections in hospitalized patients, osteomyelitis and malignant external otitis in diabetics, and folliculitis associated with hot -tub use.
Diphtheria toxin which acts by the same mechanism as Pseudomonas
Shiga-like toxin of Enterohemorrhagic Escherichia Coli (EHEC) and Shiga toxin of Shigella spp. that act to degrade 28S rRNA and prevent binding of tRNA to the 60S ribosome subunit. EHEC and Shigella spp. cause bloody diarrhea or dysentery.

              B. Disrupts cytoskeleton by depolymerizing actin filaments

              Toxin B (cytotoxin) of Clostridium difficile acts by disrupting the cytoskeleton by depolymerizing actin filaments, which results in cell death of the intestinal mucosae. This leads to pseudomembranous colitis and watery diarrhea.

              C. Increases cAMP

              Exotoxins that act to increase cAMP include pertussis toxin of Bordetella pertussis, edema and lethal factor of Bacillus anthracis, cholera toxin of Vibrio cholera, diarrheagenic enterotoxins of Bacillus cereus, and the enterotoxin of Campylobacter jejuni. Bordetella pertussis is the causative agent in whooping cough. Bacillus anthracis infections typically present with necrotic pustules in cutaneous anthrax, bacteremia and meningitis in inhalation anthrax, and dysentery in alimentary tract anthrax. Vibrio cholera, Bacillus cereus, and Campylobacter jejuni are responsible for gastrointestinal infections.

              D. Acts as a protease

              Exotoxins that function as proteases include botulinum toxin of Clostridium botulinum, tetanus toxin of Clostridium tetani, lethal factor of Bacillus anthracis, exfoliatin of Staphylococcus aureus, and exotoxin B of Streptococcus pyogenes. Clostridium botulinum is responsible for botulism, which presents with bilateral cranial neuropathies and symmetric descending weakness. Clostridium tetani causes excessive muscle contraction, most commonly known as tetanus. Bacillus anthracis infections typically present with necrotic pustules in cutaneous anthrax, bacteremia and meningitis in inhalation anthrax, and dysentery in alimentary tract anthrax. The exfoliatin toxin of Staphylococcus aureus is responsible for the cleavage of desmogleins causing scalded skin syndrome, a typically non-fatal disease of newborns. Exotoxin B of Streptococcus pyogenes is responsible for necrotizing fasciitis, another skin and soft tissue infection, but one that usually follows fascial planes.This is in contrast to the patient’s clostridial myonecrosis, which enters deep as evidenced by muscle necrosis and elevated creatinine kinase.
33
Q
A 10-year-old boy presents in the clinic with an itchy, red rash on his arm. He has been at summer camp for the past two weeks where he spent time swimming in the lake, hiking in the woods, and making pottery. His past medical history is notable for mild persistent asthma and his immunizations are up to date. On physical exam, he has a linear erythematous rash with weeping blisters on his forearm. The rest of his physical exam is unremarkable.
What hypersensitivity rxn is this?
A. Type i
B. Type ii
C. Type iii
D. Type iv
A

D. Type iv
The boy in the vignette has a pruritic linear rash with blistering that is characteristic of poison ivy, oak, or sumac. These plants produce a resin called urushiol that causes a type IV hypersensitivity reaction. Urushiol is a hapten that is phagocytosed by Langerhans cells in the skin and presented to T-lymphocytes in regional lymph nodes. These T-lymphocytes become activated, expand in the circulation, and migrate to the site of exposure. Re-exposure to urushiol triggers the release of cytokines, which leads to local tissue damage within 12-48 hours.

A. Type I

              Type I hypersensitivity reactions are IgE-mediated. Examples include anaphylaxis and atopy (allergic rhinitis, eczema, hives).

              B. Type II

              Type II hypersensitivity reactions are cytotoxic, antibody-mediated reactions. Examples include hemolytic anemia, pernicious anemia, ITP, acute hemolytic transfusion reaction, rheumatic fever, Goodpasture’s disease, bullous pemphigoid, pemphigus vulgaris, Grave’s disease, myasthenia gravis, and erythroblastic fatalis.

              C. Type III

              Type III hypersensitivity reactions are immune complex-mediated reactions. Examples include systemic lupus erythematosus, rheumatoid arthritis, polyarteritis nodosa, poststreptococcal glomerulonephritis, serum sickness, Arthus reaction, and hypersensitivity pneumonitis.
34
Q

An 11-year-old boy presents to the emergency department with progressive neck and back pain, myalgias, and weakness of his right leg over the past 2 days. In addition, his parents report that he was sick for a few days about 1 week ago with headache, sore throat, fever to 38.33°C(101°F), and fatigue, but he actually felt better for an intervening 3 days before the onset of his current symptoms. The patient and his family just immigrated 3 weeks ago to the United States from a rural area of Pakistan. The parents state that several people in their village had symptoms similar to those of the patient’s initial illness. The patient has never received any vaccines. Physical examination reveals extreme weakness of the right lower extremity, particularly of proximal muscles, with decreased deep tendon reflexes; sensory exam is normal. The patient has normal mental status with a normal cognitive exam. Magnetic resonance imaging (MRI) of the spine reveals evidence of destruction of anterior horn cells. Evaluation of cerebrospinal fluid (CSF) shows a pleocytosis, and polymerase chain reaction (PCR) testing on CSF is positive for a certain single-stranded RNA virus.

What is the route of transmission?

A. Transmission by fecal oral route
B. Transmission via respiratory secretions
C. Transmission from mother to infant (vertical transmission)
D. Transmission via sexual contact
E. Transmission via arthropods

A

A. Transmission by fecal oral route
This patient’s signs and symptoms, exposure history, physical examination findings, and neuroimaging findings are all most consistent with poliovirus infection. About 4% to 8% of affected people develop a minor illness that is indistinguishable from other viral illnesses and resolves within a week. Involvement of the central nervous system occurs in 1% to 2% of people, manifesting as non-paralytic aseptic meningitis, usually after a symptom-free period lasting for a few days. Fewer than 1% of infections result in paralysis, which generally begins 1 to 10 days after the initial minor illness and progresses over 2 to 3 days. Poliovirus causes destruction of motor neurons in the anterior horns of the spinal cord, causing neck, back, and muscle pain and asymmetric muscle weakness. Proximal muscles are typically affected more than distal, and lower limbs more than upper. Poliovirus infection is diagnosed based on clinical presentation and on detection of the virus in CSF by PCR or in throat or stool culture. The virus is acquired by fecal-oral transmission. Although it has been eradicated from most of the world, it remains endemic in 3 countries—Afghanistan, Nigeria, and Pakistan. Two vaccines are available: a live attenuated oral vaccine (OPV) and an inactivated vaccine (IPV) administered intramuscularly or subcutaneously.

B. Transmission via respiratory secretions

              Transmission via respiratory secretions is an important route of spread for several viral infections, including herpes simplex virus 1 (HSV-1), varicella zoster virus (VZV), Epstein-Barr virus (EBV), measles, influenza, and adenovirus.

              C. Transmission from mother to infant (vertical transmission)

              Viruses that spread via transmission from mother to infant (vertical transmission) include HSV-2, cytomegalovirus (CMV), rubella, hepatitis B virus (HBV), and human immunodeficiency virus (HIV).

              D. Transmission via sexual contact

              Viruses transmitted via sexual contact include HSV-2 and (less commonly) HSV-1, CMV, human herpesvirus-8 (HHV-8), HBV, HIV, and human papillomaviruses 6 and 11 (HPV-6 and -11).

              E. Transmission by arthropods

              Viruses transmitted by arthropods (insects) are referred to as arboviruses. They include mosquito-borne viruses, such as yellow fever virus, dengue virus, St. Louis encephalitis virus, West Nile virus, chikungunya virus, and Eastern equine encephalitis virus, as well as tick-borne viruses, such as Crimean-Congo hemorrhagic fever virus and tick-borne encephalitis viruses.
35
Q

A 72-year-old retired surgeon presents to the neurology clinic with progressive cognitive decline over one year. His wife notes that he consistently forgets where he has placed things and has started forgetting names of his friends whom he has known for years. In addition, she states he will occasionally see people or “shadows” in the bedroom when there are none present. His only medications are metoprolol and warfarin for atrial fibrillation. His vitals are within normal limits. On cardiovascular examination, the patient has an irregular rhythm; however, there are no murmurs or gallops. A neurologic exam is notable for normal strength, intact sensation, and bilateral upper extremity rigidity.
Most likely diagnosis?

A. Alzheimer's dementia
B. Lewy body dementia
C. Wilson's disease
D. Pick's disease
E. Multi-infarct dementia
A

B. Lewy body dementia
The patient most likely suffers from Lewy body dementia. In addition to dementia, the disease is characterized by extrapyramidal signs (e.g. rigidity) that are reminiscent of Parkinson disease. Patients with Lewy body dementia often experience visual hallucinations. Lewy bodies are intracytoplasmic, eosinophilic inclusions with a dense core of ɑ-synuclein surrounded by a pale halo.

A. Alzheimer’s dementia

              Alzheimer’s disease is the most common cause of dementia in the elderly. It is a less likely diagnosis in this patient given the extrapyramidal signs and visual hallucinations, which are more commonly found in Lewy body dementia.

              C. Wilson’s disease

              Wilson disease (hepatolenticular degeneration) is an autosomal recessive disorder characterized by deficient excretion and subsequent accumulation of copper in several organs. Patients may present with signs and symptoms of jaundice, dysarthria, clumsiness, tremor, drooling, gait disturbance, malaise, and arthralgia. Copper deposition in the basal ganglia can result in Parkinsonian symptoms. Although the patient presents with dementia and Parkinsonism, Wilson disease is unlikely due to the lack of other defining characteristics of the disease in this patient.

              D. Pick’s disease

              Pick disease is a frontotemporal dementia characterized by early behavioral changes, frontal release signs, and difficulties with language. While patients with Pick disease can demonstrate Parkinsonian signs, the presence of visual hallucinations is more consistent with Lewy body dementia.

              E. Multi-infarct dementia

              Multi-infarct dementia, or vascular dementia, is the second most common cause of dementia in the elderly. It is caused by cerebral atherosclerosis and vascular insult. The patient’s history of atrial fibrillation and irregular heart rhythm on exam are risk factors for a cerebrovascular accident and create concert for vascular dementia. While multi-infarct dementia can present with a variety of signs and symptoms, Parkinsonian features in the presence of visual hallucinations is more indicative of Lewy body dementia.
36
Q

A 24-month-old patient is evaluated in the Emergency Room after she is found playing with some pills that she found at her aunt’s house. The aunt has a past medical history of atrial fibrillation and diabetes mellitus type II. Her vital signs include: Temperature is 37.1°C (98.78°F), heart rate is 55 beats per minute, blood pressure is 60/40 mmHg, and respiratory rate is 25 breaths per minute. On physical exam, she appears lethargic. Her heart rate is bradycardic. Her lungs are clear to auscultation bilaterally. Her extremities are cool and mottled and she has weak peripheral pulses. A fingerstick blood glucose is 95, INR is 1.0, and her electrocardiogram shows sinus bradycardia.
What is the most appropriate treatment for this child?

A. Glucagon
B. Albuterol
C. Naloxone
D. Vit K
E. Dextrose
A

A. Glucagon
The child likely ingested a beta-blocker causing her bradycardia, hypotension, lethargy, and weak pulses. High-dose glucagonis used to treat bradycardia and hypotension in cases of beta-blocker ingestion.
B. Albuterol

              Bronchospasm is a potential side effect of beta-blockers, treated with inhaled beta-agonists, however the patient in this vignette does not have any wheezing.

              C. Naloxone

              Naloxone is an opioid receptor antagonist that can be used to treat opiate overdose.

              D. Vitamin K

              Vitamin K is used to reverse the effects of warfarin, however it is not indicated in this patient who has a normal INR.

              E. Dextrose

              Dextrose is not indicated, as her D-stick is normal.
37
Q

A 32-year-old woman develops the sudden onset of global aphasia and right-sided hemiparesis. She is brought to the emergency department where she is found to suffer from a left middle cerebral artery infarct. Diagnostic imaging identifies a carotid dissection as the likely cause of her stroke. Additionally, cerebrovascular imaging reveals a “string of beads” appearance of the cerebral blood vessels.

A. Fibromuscular dysplasia
B. Malignancy
C. Lupus
D. Giant cell arteritis

A

The patient suffers from fibromuscular dysplasia, a vascular condition that results in the narrowing of the renal arteries resulting in hypertension. Also, it causes a narrowing of the carotid artery that can cause impaired blood flow. Additionally, the condition predisposes patients to spontaneous carotid dissections that can result in ischemic stroke. The classic appearance on angiogram is the “string of beads” finding.

38
Q

A 44-year-old man is brought to the emergency department by ambulance after being found down at the grocery store. An emergent CT scan of the brain demonstrates multiple large intraparenchymal hematomas in the basal ganglia and subcortical white matter. The patient has no significant past medical history, and there are no external signs of trauma to his head. He was seen by his primary care physician two weeks prior to presentation for a routine physical, and he reported no symptoms at that time. Vital signs are within normal limits. Physical exam in the ED demonstrates extensive palpable purpura over his lower extremities bilaterally, but is otherwise unremarkable. A complete blood count and coagulation labs reveal the following:

		Leukocyte count
		7,000/mm3

		Red blood cell count
		4.5 million/mm3

		Hemoglobin
		13.8 g/dL

		Hematocrit
		38.4%

		Platelet count
		2,000/uL

		Prothrombin time
		12 seconds

		Partial thromboplastin time (activated)
		32 seconds

A basic metabolic panel reveals normal renal function. An ANA is obtained and is negative. A peripheral smear demonstrates large platelets but no schistocytes. The patient is treated with intravenous immunoglobulin (IVIG), and the platelet count begins to rise over the course of the following three days.

Diagnosis?

A. Non-Hogkin's Lymphoma
B. SLE
C. ITP
D. TTP
E. HIV
A

C. idiopathic thrombocytopenia purpura (ITP)
This patient is most likely suffering from idiopathic thrombocytopenia purpura (ITP). ITP is an autoimmune disorder characterized by development of IgG antibodies against the GPIIb/IIIa receptor on platelets. Notable signs and symptoms of ITP include the sudden appearance of a petechial rash, easy bleeding, or bruising. Notable laboratory findings include thrombocytopenia, large platelets on peripheral smear, and normal PT/PTT times. This patient is suffering from an intracranial hemorrhage, a potentially fatal complication of ITP. When platelet levels fall to dangerously low levels, spontaneous bleeding in different organs can occur with minimal or no trauma. In severe cases of ITP, spontaneous intracranial bleeding can occur. The first lines of treatment include high-dose steroids and intravenous immunoglobulin (IVIG).

A. Non-Hodgkin’s lymphoma

              Hematologic malignancies such as non-Hodgkin's lymphoma can occasionally present with acute thrombocytopenia. However, this patient has no history of constitutional symptoms suggestive of lymphoma, making this diagnosis unlikely.

              B. Systemic lupus erythematosus

              Systemic lupus erythematosus (SLE) is an autoimmune disorder than can present with multiple organ system dysfunction, including hematologic synthetic dysfunction and thrombocytopenia. The most common presenting symptoms and signs are a characteristic rash and renal dysfunction, as well as constitutional symptoms such as fever, weight loss, and fatigue. This patient has no symptoms suggestive of lupus, and moreover, he has a negative ANA, making this diagnosis unlikely.

              D. Thrombotic thrombocytopenic purpura (TTP)

              Thrombotic thrombocytopenic purpura (TTP) is a rare cause of acquired thrombocytopenia. Unlike ITP, TTP also causes microangiopathic hemolytic anemia, renal failure, encephalopathy, and fever. It is treated with plasmapheresis. As this patient has a normal hematocrit, normal renal function, and there are no schistocytes on the peripheral smear, this diagnosis is unlikely.

              E. Human immunodeficiency virus

              Human immunodeficiency virus (HIV) is a rare cause of thrombotic thrombocytopenic purpura (TTP). This patient has no risk factors for or history of HIV, and furthermore, his presentation is more consistent with ITP than TTP, making this diagnosis unlikely.
39
Q

A 15-year-old male presents to the emergency department during the winter months complaining of eye pain and blurry vision. He saw his primary care physician about a week ago with fever and eye redness. At the time he also hadmalaise, headache, muscle aches, and a sore throat all of which have resolved. He was tested forinfluenza and group A streptococcus which were negative. He was treated with ibuprofen.He has no significant past medical history and takes no other medications. He notes that he is part of a dance troupe that recently participated in a major festival ten days ago and many of the other performers now have similar symptoms.Vital signs are normal.On physical examination, there are several corneal opacities. Pupils are equal round and reactive to light. Extraoccular muscles are intact and there is no pain associated with their movement. Tonsils are 2+ without exudates and there is mild erythema of the posterior oropharynx. There are 1-2 cm tender, mobile lymph nodes in the preauricular areas. There are no skin rashes. The rest of the physical exam is unremarkable.
Diagnosis?

A. Epidemic keratoconjunctivitis
B. Orbital cellulitis
C. Preseptal cellulitis
D. Allergic conjunctivitis
E. Acute hemorrhagic conjunctivitis
F. Herpes simplex virus keratitis
G. Bacterial conjunctivitis
A

A. Epidemic keratoconjunctivitis
Epidemic keratoconjunctivitis is a more serious form of conjunctivitis caused by certain strains of adenovirus. It ischaracterized by fever, eye pain and inflammaiton, and preauricular lymphadenopathy, that isfollowed by the development of painful corneal opacities. It is highly contagious and often occurs in outbreaks.Symptoms are self-limited and rarely result in permanent visual deficits.

B. Orbital cellulitis

              Orbital cellulitis is an infection involving the fat and ocular musclesof the orbit. Patients often have sinusitis prior to the development of this condition. Symptoms include eyelid swelling, pain or limitation of eye movement, proptosis, and visual loss.Orbital cellulitis is an emergency that requires intravenous antibiotics.

              C. Preseptal cellulitis

              Preseptal cellulitis is an infection of the soft tissues anterior to the orbital septum.Patients have eyelid swelling without pain or limitation of eye movement, proptosis, or visual loss.

              D. Allergic conjunctivitis

              Allergic conjunctivitis is caused by airborne allergens that bind to specific IgE in the eye leading to local mast cell degranulation and the release of histamine among other chemical mediators. Symptoms are also bilateral and include redness, watery discharge, and itching.Patients often have a history of seasonal allergies or specific allergies (e.g. cat dander).

              E. ​Acute hemorrhagic conjunctivitis (AHC)

              AHC is a highly contagious ocular infection characterized by pain, lid edema, and subconjunctival hemorrhage that is caused by entero- and coxsackie viruses.It is self-limited and rarely leads to permanent visual impairment.

              F. Herpes simplex virus (HSV) keratitis

              HSV keratitis has an acute onset. Symptoms include pain, blurry vision, and discharge. Physical examination is notable for chemosis, conjunctivitis, and decreased corneal sensation. There are characteristic dendritic lesions on the cornea in contrast to the corneal opacities seen with EKC. Blindness results without appropriate treatment.

              G. Bacterial conjunctivitis

              Bacterial conjunctivitis is typically caused by Staphylococcus aureus, Streptococcus pneumoniae, Haemophilus influenzae, or Moraxella catarrhalis. Like viral conjunctivitis it presents with pain and discharge. In both conditions, patient report discharge and that the eyes are "stuck-closed" in the morning. However, in bacterial conjunctivitis, the discharge is not just in the morning, but continues throughout the day. The discharge in bacterial conjunctivitis is typically thick and yellow, while inviral or allergic conjunctivitis the discharge is thin and watery.
40
Q

A 2.5-year-old female is brought to the emergency room after her mother found her with an open bottle of pills belonging to the child’s aunt. Sheis unsure what kind of pills theywere orhow many the child consumed. However, shethe aunt recently gave birth to a healthy infant. Themother reports that her daughter had multiple episodes of brown-black emesis almost immediately and vomited up some of the pills. She thenappeared completely well so the mother did not seek care. Six hours later the child seemed sleepier than normal and was breathing quickly so the mother called 9-1-1. The childhas no significant past medical history and her immunizations are up-to-date.Vital signs are as follows: temperature 37°C (98.6°F), heart rate 150/min, blood pressure 76/38 mmHg, respiratory rate 30/min, and oxygen saturation 97% on room air. On physical exam,the child islethargicand tachypneic, heart rate is tachycardic, lungs are clear to auscultation bilaterally. Her abdomen is diffusely tender to palpation. Extremities are cool and capillary refill is5 seconds.
Best treatment for patient’s condition?

A. Deferoxime
B. Glucagon
C. Hemodialysis
D. N-Acetylcysteine
E. Fomepizole
A

A. Deferoxime
The patient in this vignette is suffering from acute iron poisoning. Soon after consuming a large dose of iron, patients develop gastrointestinal symptoms including nausea, vomiting, and abdominal pain. Some patients then experience a latent phase where they appear relatively well. This is soon followed by shock and metabolic acidosis,hepatotoxicity, and bowel obstruction. The patient in this vignette has physical exam signs and symptoms that are concerning for shock and metabolic acidosis include tachypnea, tachycardia, and poor perfusion. Prenatal vitamins and pure iron preparations contain more iron than other preparations and are often candy-colored and coated in sugar making them appealing to young children. Given that the aunt recently gave birth it is likely that the pills were prenatal vitamins or iron. The treatment for acute iron poisoning is gastric lavage and deferoxime.

B. Glucagon

              Glucagon is indicated in the treatment of severe beta-blocker poisoning, which presents with bradycardia and hypotension.

              C. Hemodialysis

              ​Hemodialysis is rarely indication in the treatment of poisonings. Some of the medications that are amenable to treatment include: barbiturates, alcohols, lithium, salicylates, and some beta blockers.

              D. N-Acetylcysteine

              N-Acetylcysteine is used to treat patients with acetaminophen toxicity. It protects the liver by restoring hepatic glutathione. Patients with acetaminophen toxicity are initially asymptomatic and then develop hepatic dysfunction leading to fulminant hepatic failure.

              E. Fomepizole

              ​Fomepizole is an alcohol dehydrogenase inhibitor used in the treatment of ethylene glycol or methanol ingestions. Patient with these ingestions present with sedation and metabolic acidosis. Methanol poisoning can cause blurry vision and blindness. Ethylene glycol poisoning can cause renal failure.
41
Q

A 24-year-old man presents to the emergency department for suicidal ideation and a plan to jump off a local bridge. On interview, he endorses poor sleep, low interest in his hobbies, low energy, and an overwhelming feeling of sadness that “just won’t leave him.” When asked how long he has been feeling this way, he says, “for at least a month, maybe more. I always start feeling this way when the government is bugging me. They keep trying to listen to my thoughts even though I call Homeland Security to tell them to leave me alone.” The patient gives permission to call his brother, who is also his roommate. His brother confirms the patient’s history, adding, “I hate to say that my brother’s crazy, but there’s no other way to describe it. The past few weeks, I feel like he’s been talking more to people that aren’t there than he is to me. He’s done this before- he gets depressed and starts hearing voices and becomes obsessed about the government being after him, which makes him even more depressed. Except there was one time, about two years ago, when he was convinced that his thoughts were being reported on the local radio station. He was pretty angry then, but definitely not depressed.” The patient denies substance use, stating that using drugs just makes him feel more paranoid.

Diagnosis?
A. Schizophrenia
B. Schizoaffective
C. Major depressive disorder with Psychotic symptoms
D. Schizophreniform
A

This patient is most likely to have schizoaffective disorder because he most often experiences psychotic symptoms along with major depressive episodes. However, he has also had psychotic symptoms in the past while he was not depressed. To be given a diagnosis of schizoaffective disorder, patients must meet the following four criteria.

		A
		An uninterrupted period of illness during which there is a major mood episode (major depressive or manic) at the same time as with the patient meets criterion A of schizophrenia (2 or more of delusions, hallucinations, disorganized speech, disorganized behavior, or negative symptoms for one month or more)

		B
		Delusions or hallucinations for two or more weeks in the absence of a major mood episode during the lifetime duration of the illness

		C
		Symptoms that meet criteria for major mood episode are present most of the time during the active disease

		D
		Psychiatric symptoms are not because of the effects of a substance or another medical condition

The patient in the vignette does meet the criteria for schizoaffective disorder. His current symptoms have lasted for a month, meet the criteria for major depressive disorder (poor sleep, low mood/sadness, decreased interest, low energy, and suicidality), and meet criterion A for schizophrenia (evidence of delusions and hallucinations reported by his brother). His brother provides evidence for criterion B- he has had a past episode of delusions without major mood symptoms. His brother also provides evidence for criterion C- most of the time when he is sick with his psychotic symptoms, he also seems to have mood symptoms. Finally, there is no mention of substance use or other medical conditions.

A. Schizophrenia

              The best diagnosis for this patient is not schizophrenia, because patients with schizophrenia should not meet criteria for a major mood episode during the majority of their psychotic episodes. Because the patient in the vignette has a history of experiencing major depression alongside most of his psychotic episodes, schizoaffective disorder is a better diagnosis.

              C. Major Depressive Disorder with Psychotic Features

              The reason the patient in the vignette does not meet criteria for major depressive disorder with psychotic features is because he has had psychotic symptoms at some point when he was not depressed. For a diagnosis of major depressive disorder with psychotic features to be appropriate, a patient must only exhibit psychotic symptoms while meeting criteria for a major depressive episode.

              D. Schizophreniform Disorder

              Because this patient experiences mood symptoms during most (but not all) of his psychotic episodes, schizoaffective disorder is a more appropriate diagnosis than either schizophrenia or schizophreniform disorder. In addition, this patient has at least a two-year history of illness, whereas a diagnosis of schizophreniform disorder should be given in cases where the duration of illness has been greater than one month but less than six months.
42
Q

A 7-day-old female infant is brought to the emergency department for excessive sleepiness. The child was born to a G3P2→3 mother via vaginal delivery. There were no complications during pregnancy or delivery and all of the prenatal testing was normal. The infant was discharged from the hospital with her mother when she was three days old. Over the past two days the infant has been feeding poorly and today was difficult to rouse from sleep. Family history is unremarkable. Vital signs are as follows: temperature 37°C (98.6°F), heart rate 160 beats/min, blood pressure 72/40 mmHg, respiratory rate 70 breaths/min, and oxygen saturation 100% on room air.On exam, she is minimally responsive. There is no heart murmur and lungs are clear to auscultation bilaterally. Her abdomen is soft, non-tender, and non-distended. There are no rashes or jaundice. You ask the nurse to draw blood and obtain a urine specimen. The nurse comments that the urine smells like burnt sugar. A complete blood cell count and differential are normal. There are 0 white blood cells in the cerebral spinal fluid and protein and glucose are normal. The urine is positive for ketones, but the leukocyte esterase and nitrites are negative.

A. Leucine
B. Homocysteine
C. Phenylalanine
D. Lactose
E. Galactose
A

A. Leucine

This is a classic description of maple syrup urine disease. In this disease there is a defect in the branched-chain α-keto acid dehydrogenase complex (BCKD). BCKD is responsible for the metabolism of the branched chain amino acids leucine, valine, and isoleucine, as well as odd-numbered fatty acids. As a result, these metabolites build up in the system. Signs and symptoms of maple syrup urine disease include encephalopathy and a characteristic “Maple Syrup” smell in the urine. BCKD requires many cofactors including vitamins B1, B2, B3, CoA (from B5) and lipoic acid. These same cofactors are required by pyruvate dehydrogenase, which converts pyruvate to acetyl-CoA.

B. Homocysteine

              A defect in the enzyme involved in homocysteine metabolism causes Homocysteinuria. Clinical features of this disorder include ectopia lentis, Marfanoid features, thromboembolism, intellectual disability, seizures, osteoporosis, and anemia.

              C. Phenylalanine

              A defect in the enzyme involved in the metabolism of phenylalanine leads to the disorder Phenylketonuria which presents with developmental delay, eczema, seizures, self-mutilation, and severe behavioral disorders. It is associated with patient's having a mousy odor.

              D. Lactose

              Lactose is a sugar, not an amino acid.

              E. Galactose

              Galactose is a sugar, not an amino acid.
43
Q

A 34 year-old man experiences the sudden onset of back pain radiating to his right anterolateral thigh while lifting weights at the gym. Three days after symptom onset, he presents to his primary care physician for evaluation. On examination he has weakness with knee extension in his right lower extremity and an absent right patellar tendon reflex.
Most likely cause of symptoms?

A. Herniated T12/L1 intervertebral disc
B. Herniated L1/L2 intervertebral disc
C. Herniated L2/L3 intervertebral disc
D. Herniated L4/L5 intervertebral disc
E. Herniated L5/S1 intervertebral disc
A

C. Herniated L2/L3 intervertebral disc
This patient is suffering from a herniated L2/L3 lumbar disc impinging on the L3 nerve root. In the lumbar spine, posterolateral disc herniation affects the nerve root corresponding to the lower vertebral level.L2-3 disc herniation typically produces pain radiating from the lower back into the anterolateral thigh. As in this case, exam will demonstrate weakness in the quadriceps muscle group manifested as impaired knee extension and a diminished patellar tendon reflex. The syndrome can be difficult to distinguish from anL3/4 disc herniation, which produces pain and numbness in theposterolateral thigh and medial shin. Exam will similarly show a weakened quadriceps and diminished patellar tendon reflex.

A. Herniated T12/L1 intervertebral disc

              L1 radiculopathy produces pain and numbness over the ipsilateral inguinal region. Weakness with hip flexion may rarely be observed, but all lower extremity deep tendon reflexes are generally intact.

              B. Herniated L1/L2 intervertebral disc

              L1/2 disc herniation produces pain in the anterior and medial aspect of upper thigh. Weakness may be appreciated with hip flexion/extension and knee extension, but the patellar tendon reflex is usually spared.

              D. Herniated L4/L5 intervertebral disc

              L4/5 disc herniation classically presents as pain in dorsum of the foot with weakness in the extensor muscles of the big toe and foot. Deep tendon reflexes will typically be intact.

              E. Herniated L5/S1 intervertebral disc

              L5/S1 herniation produces pain in the lateral aspect of the foot; exam is notable for a diminished or absent Achilles tendon reflex.
44
Q

A 2-year-old presents to the emergency department with fever in July. Two days ago she had a fever of 40°C (104°F), sore throat, anorexia, and diarrhea. This morning she also started to complain of a headache and was more tired than usual. She is otherwise healthy and her immunizations are up-to-date. Her temperature is 39°C (102.2°F), heart rate is 100 beats per minute, blood pressure is 105/75 mmHg, respiratory rate is 26 breaths per minute, and oxygen saturation is 97% on room air. On physical exam, her tympanic membranes are mildly erythematous and her oropharynx is clear. There is pain with flexion of her neck. There is no murmur. Lungs are clear to auscultation bilaterally. Abdomen is soft non-tender, non-distended, with no hepatosplenomegaly. She has a blanching maculopapular rash on her trunk. A complete blood count, blood cultures, urine cultures, and cerebral spinal fluid (CSF) studies are obtained. The CSF is clear. There are 0 red blood cells per mL, 700 white blood cells per mL with 75% lymphocytes, a glucose of 80 mg/dL, and protein of 45 mg/dL. Gram stain is negative.

Most likely pathogen?
A. Neisseria meningitidis
B. HSV
C. Enterovirus
D. Strep pneumoniae
E. Rubulavirus
A

C. Enterovirus
Enterovirus causes 90% of all cases of aseptic meningitis in immunized children. Infections are more common in the summer. The child in the vignette has evidence of enteroviral meningitis including a prodrome of fever, upper respiratory symptoms, and gastrointestinal symptoms followed by the onset of headache, neck stiffness, and photophobia. You can also see a nonspecific maculopapular rash. Her cerebrospinal fluid studies are consistent with a viral meningitis: clear fluid, normal red blood cell count (normal is

45
Q
A 10-year-old boy is evaluated in the emergency department after recently being rescued from a national park. He was on a camping trip when he became separated from the rest of his family. He was located by a park ranger four days later. During this time he subsisted on one granola bar and some nuts and berries that he foraged. Vital signs include temperature 36°C (96.8°F), heart rate 64 beats/min, blood pressure 80/56 mmHg, respiratory rate 12 breaths/min, and oxygen saturation 100% on room air. On physical exam, he appears thin and his muscles are weak. Heart, lungs, and abdomen are normal. He has numerous bruises and abrasions on his skin. During the period of starvation experienced by this young boy, protein primarily from skeletal muscle was catabolized resulting in the accumulation of nitrogenous waste.
In what form was the excess nitrogen transported from his skeletal muscle tissue to his liver for excretion?
A. Alanine
B. Urea
C. Ammonia
D. Pyruvate
E. Albumin
F. alpha-ketoglutarate
A

A. Alanine
NH3 (ammonia) is produced during protein catabolism. NH3 is toxic and must be excreted. NH3 is taken to the liver, where it is then converted to urea for excretion. In muscle tissue, NH3 is transferred from glutamate to pyruvate in a transamination reaction, forming alanine and α-ketoglutarate. Alanine exits the muscle and travels in the blood to the liver, where it is taken up and undergoes the same transamination reaction in reverse, regenerating pyruvate and glutamate. The pyruvate is used in gluconeogenesis and the glutamate provides one of the NH3 groups that form the molecule urea.

B. Urea

              Urea is generated in the liver, not in skeletal muscle.

              C. Ammonia

              NH3 is transferred from glutamate to pyruvate in a transamination reaction, forming alanine and α-ketoglutarate; alanine transports nitrogen from skeletal muscle to the liver.

              D. Pyruvate

              Pyruvate is used in skeletal muscle then regenerated in the liver, but does not transport nitrogen between skeletal muscle and the liver.

              E. Albumin

              Albumin is a serum protein, but it does not transport nitrogen from skeletal muscle to the liver.

              F. α-ketoglutarate

              NH3 is transferred from glutamate to pyruvate in a transamination reaction that forms alanine and α-ketoglutarate. However, alanine not α-ketoglutarate transports nitrogen from skeletal muscle to the liver.
46
Q
A 10-year-old boy is evaluated in the emergency department after recently being rescued from a national park. He was on a camping trip when he became separated from the rest of his family. He was located by a park ranger four days later. During this time he subsisted on one granola bar and some nuts and berries that he foraged. Vital signs include temperature 36°C (96.8°F), heart rate 64 beats/min, blood pressure 80/56 mmHg, respiratory rate 12 breaths/min, and oxygen saturation 100% on room air. On physical exam, he appears thin and his muscles are weak. Heart, lungs, and abdomen are normal. He has numerous bruises and abrasions on his skin. During the period of starvation experienced by this young boy, protein primarily from skeletal muscle was catabolized resulting in the accumulation of nitrogenous waste.
In what form was the excess nitrogen transported from his skeletal muscle tissue to his liver for excretion?
A. Alanine
B. Urea
C. Ammonia
D. Pyruvate
E. Albumin
F. alpha-ketoglutarate
A

A. Alanine
NH3 (ammonia) is produced during protein catabolism. NH3 is toxic and must be excreted. NH3 is taken to the liver, where it is then converted to urea for excretion. In muscle tissue, NH3 is transferred from glutamate to pyruvate in a transamination reaction, forming alanine and α-ketoglutarate. Alanine exits the muscle and travels in the blood to the liver, where it is taken up and undergoes the same transamination reaction in reverse, regenerating pyruvate and glutamate. The pyruvate is used in gluconeogenesis and the glutamate provides one of the NH3 groups that form the molecule urea.

B. Urea

              Urea is generated in the liver, not in skeletal muscle.

              C. Ammonia

              NH3 is transferred from glutamate to pyruvate in a transamination reaction, forming alanine and α-ketoglutarate; alanine transports nitrogen from skeletal muscle to the liver.

              D. Pyruvate

              Pyruvate is used in skeletal muscle then regenerated in the liver, but does not transport nitrogen between skeletal muscle and the liver.

              E. Albumin

              Albumin is a serum protein, but it does not transport nitrogen from skeletal muscle to the liver.

              F. α-ketoglutarate

              NH3 is transferred from glutamate to pyruvate in a transamination reaction that forms alanine and α-ketoglutarate. However, alanine not α-ketoglutarate transports nitrogen from skeletal muscle to the liver.
47
Q

You are evaluating a 14-month old boy in the emergency department who presents with a fever. His past medical history is notable for recurrent urinary tract infections and skin abscesses since six months of age for which he takes trimethoprim-sulfamethoxazole daily. His maternal uncle also suffered from recurrent infections and died at the age of 4. On physical exam his temperature is 39.5ºC (103.1ºF), heart rate 120/min, blood pressure is 86/49, respiratory rate is 34/min, and oxygen saturation is 90% on room air. He appears ill and is using accessory muscles to breath. A chest x-ray shows a right lower lobe consolidation. He is admitted to the hospital for further treatment and evaluation. Laboratory investigations are notable for anemia, hypergammaglobulinemia, and neutropenia.
Which of the following defects is the most likely cause of this child’s recurrent infections?
A. Deficient superoxidase production by macrophages and neutrophils
B. A defect in microtubule polymerization
C. A defect in leukocyte chemotaxis
D. A DNA repair defect
E. Decreased levels of immunoglobulin
F. Inability to form membrane attack complexes

A

A. Deficient superoxidase production by macrophages and neutrophils
The child in this vignette has recurrent urinary tract infections and skin abscesses despite daily antibiotic prophylaxis which is concerning for an immunodeficiency disorder. The fact that a maternal uncle also had recurrent infections is concerning for an X-linked recessive disorder. His labs are notable for anemia, hypergammaglobulinemia, and neutropenia. These findings are highly suggestive of Chronic granulomatous disease (CGD), an X-linked or autosomal recessive disorder that is caused by a deficiency in the production of superoxide by polymorphonuclear cells and macrophages. Patients with CGD present with recurrent skin, pulmonary, gastrointestinal, urinary tract, and bone infections with catalase + organisms. A positive nitroblue tetrazolium test is diagnostic

B. A defect in microtubule polymerization

              A defect in microtubule polymerization which causes a defect in neutrophil chemotaxis is the cause of Chediak-Higashi syndrome. This is an autosomal-recessive immunodeficiency disorder characterized by oculocutaneous albinism, neuropathy, and neutropenia.

              C. A defect in leukocyte chemotaxis

              A defect in leukocyte chemotaxis is associated with leukocyte adhesion deficiency, which presents with recurrent skin, mucosal, and pulmonary infections as well as omphalitis in the newborn period. Wounds typically have minimal pus and inflammation and laboratory investigations show a leukocytosis.

              D. A DNA repair defect

              A DNA repair defect is associated with many disorder including the immunodeficiency disorder Ataxia-Telangiectasia, which presents with oculocutaneous telangiectasias, progressive cerebellar ataxia, and recurrent infections. 

              E. Decreased levels of immunoglobulin

              Decreased levels of immunoglobulin are associated with Bruton’s X-linked agammaglobulinemia, which presents with recurrent infections and absent tonsils.

              F. Inability to form membrane attack complexes

              The inability to form membrane attack complexes is characteristic of terminal complement deficiency (C5-C9), which presents with recurrent meningococcal or gonococcal infections and rarely, lupus or glomerulonephritis.
48
Q
A 27-year-old woman presents with fullness, diminished hearing, and intermittent drainage in her right ear. Her past medical history is notable for common variable immunodeficiency for which she receives regular IV immunoglobulin infusions. Prior to her immunodeficiency diagnosis, she had multiple sinus infections and several episodes of acute otitis media. Examination of the right ear reveals a perforated tympanic membrane with a cystic mass protruding into the external auditory meatus. The left ear is normal.
Diagnosis?
A. Craniopharyngioma
B. Otosclerosis
C. Ethesioneuroepithelioma
D. Cholesteatoma
E. Glomus tumor
A

D. Cholesteatoma
The patient in this vignette has a lesion in her ear that is most likely a cholesteatoma, a destructive and expanding mass of keratinizing squamous epithelium within the middle ear. Cholesteatomas typically present with chronic otorrhea, ear fullness, and unilateral conductive hearing loss. They can be congenital or occur following an episode of otitis media that causes rupture or retraction of the tympanic membrane. Treatment consists of surgical resection.

A. Craniopharyngioma

              Craniopharyngiomas are benign intracranial tumors derived from remnants of the surface ectoderm from Rathke’s pouch. They are generally suprachiasmatic in location and present with bitemporal hemianopia and endocrine abnormalities due to pressure on the anterior pituitary.

              B. Otosclerosis

              Otosclerosis is characterized by abnormal bone deposition at the footplate that leads to fixation of the stapes at the oval window, preventing normal vibration. It is more common in middle-aged Caucasian women and typically presents as progressive bilateral conductive hearing loss. Treatment consists of amplification with hearing aids or surgical repair by stapedectomy.

              C. Ethesioneuroepithelioma

              Ethesioneuoepithelioma is an aggressive and malignant neoplasm derived from olfactory epithelium. While sinus invasion and expansion to the middle ear is possible, it is not the most likely etiology in this patient.

              E. Glomus tumor

              Glomus tumor is a neuroendocrine tumor that arises from the adventitia of the jugular bulb or the neural plexus within the middle ear space. They are more common in women in their forties and present with pulsatile tinnitus and hearing loss. On examination, a pulsatile reddish-blue mass may be seen behind an intact tympanic membrane.
49
Q
A 26-year-old man presents to his primary care physician at the request of his girlfriend, who is overheard in the waiting room saying, "if you don't get help, I'm going to leave you. This is last the straw." When asked the reason for his visit, the patient reports that his girlfriend is worried because he "acts like a daredevil" when he has been drinking alcohol. He works as a roofer, and in the past year, has suffered two broken bones from jumping off of roofs after drinking. When asked if he has ever tried to cut down, the patient replies, "Not really. In fact, I notice I need to drink more and more just to get buzzed. I've pretty much stopped going fishing with my buddies because I just can't afford it anymore." The patient has normal vital signs and a normal physical exam.
A. Alcohol use disorder
B. Alcohol withdrawal
C. Alcohol abuse disorder
D. Social Drinking
A

Under the DSM-5, alcohol use and alcohol dependence have essentially been combined into one diagnosis called alcohol use disorder. In order to be diagnosed with alcohol use disorder, a patient must have two out of the following eleven criteria, occurring over a twelve-month period.

DSM-5 Alcohol Use Disorder Criteria

		1
		Alcohol often taken in larger amounts or over a longer period of time than intended

		2
		Persistent desire or unsuccessful attempts to cut down/control alcohol use

		3
		Significant amount of time spent obtaining alcohol, using it, or recovering from alcohol use

		4
		Craving or strong desire to use alcohol

		5
		Alcohol use resulting in failure to fulfill major obligations at work, school, or home

		6
		Continued alcohol use despite persistent/recurrent social/interpersonal problems resulting from use

		7
		Important social, occupational, recreational activities given up because of alcohol use

		8
		Recurrent alcohol use when it is physically hazardous

		9
		Alcohol use continued in the setting of having a known physical or psychological problem that is worsened by alcohol use

		10
		Tolerance- using more alcohol to obtain the same effect, or using the same amount with diminished effect

		11
		Withdrawal

The patient in the vignette meets four criteria for a substance use disorder, when only two are needed to make the diagnosis. He exhibits interpersonal problems exacerbated by his alcohol use (girlfriend threatening to leave him), use of alcohol in physically hazardous situations (jumping off of roofs after drinking), tolerance (needing to drink more to have the same effect), and reduction of recreational activities (fishing) as a result of alcohol use.

B. Alcohol Withdrawal

              The patient in the vignette has normal vital signs and a normal physical exam, which would not be expected in a patient with active withdrawal. If the patient was diaphoretic, tremulous, hypertensive, or tachycardic, withdrawal would be more likely.

              C. Alcohol Abuse Disorder 

              With the DSM-5, the diagnoses of "alcohol abuse" and "alcohol dependence" were eliminated, and essentially both combined under the new diagnosis of alcohol use disorder. Thus, the most current appropriate diagnosis for this patient would be alcohol use disorder rather than alcohol abuse disorder or alcohol dependence.

              D. Social Drinking

              The patient in the vignette meets criteria for an alcohol use disorder- he describes increased tolerance and giving up recreational activities because of alcohol use, and there is evidence that he is having interpersonal problems (fighting with his girlfriend) related to his alcohol use. Thus, it would not be appropriate to assume that this patient is simply engaging in "social drinking".
50
Q

A one-month old girl is hospitalized for failure to thrive. She is noted to have ambiguous genitalia on exam and vital signs reveal hypertension. Labs are notable for hypernatremia, hypokalemia, low cortisol, high ACTH, and elevated testosterone.

Most likely Diagnosis?
A. 21 OH deficiency
B. 11 beta OH deficiency
C. 17 lyase deficiency

A

B. 11 beta OH deficiency

11-beta-hydroxylase deficiency is the second most common form of congenital adrenal hyperplasia and involves the same pathway as 21-hydroxylase deficiency. While the aldosterone precursor that accumulates with 21-OH deficiency does not have any hormonal activity, the block in 11-hydroxyase allows for 11-deoxycorticosterone to be made, which has a weak mineralocorticoid effect. This causes, salt and water retention.Therefore it clinically it presents with hypertension and ambiguous female genitalia or precocious male puberty (from excess androgens). Sodium is high and potassium is low.

51
Q

A 62-year-old man who is a circus performer fell 20 feet off a stage and landed on a chair, striking his head and abdomen. He regained consciousness after 15 seconds and immediately began complaining of severe, diffuse abdominal pain. He has a history of myocardial infarction four months ago, for which he underwent percutaneous coronary intervention with the placement of a drug-eluting stent to the left circumflex artery. Current medications include acetylsalicylic acid, metoprolol, lisinopril, and clopidogrel. He arrives at the emergency department hypotensive and tachycardic. Focused assessment with sonography for trauma (FAST) scan is positive for free fluid in the peri-hepatic, hepatorenal, and perisplenic regions. He is stabilized with IV fluids and blood, and then brought emergently to the operating room for an exploratory laparotomy. Intra-operatively, there is a grade IV laceration of the spleen, requiring splenectomy. There is also evidence of several large liver lacerations. In order to temporarily achieve hemorrhage control, a Pringle maneuver(clamping of the portal triad) is performed. Liver hemorrhage persists.
What anatomical structure is responsible for the persistent hemorrhage?

A. Common bile duct
B. Portal vein
C. Hepatoduodenal ligament
D. Common Hepatic artery
E. Hepatic vein
A

E. Hepatic Vein
What anatomical structure is responsible for the persistent hemorrhage?
The portal triad consists of the common hepatic artery, the portal vein, and the common bile duct. The Pringle maneuver, as described, consists of placing a non-crushing vascular clamp across the hepatoduodenal ligament (containing the portal triad) to stop flow through the hepatic artery and portal vein. This can control liver hemorrhage in most cases. The hepatic artery arises from the systemic arterial supply to the abdominal organs, and the portal vein collects venous blood from the intestines for processing in the liver.

If bleeding persists despite the Pringle maneuver, other sources of hemorrhage must be considered, such as the hepatic vein. The hepatic veins carry deoxygenated blood from the liver into the inferior vena cava (systemic circulation). They arise from the central vein at the center of each hepatic lobule. A retro-hepatic inferior vena cava injury could also be implicated if hemorrhage persists following the Pringle maneuver.

52
Q

A 75-year-old male presents to his primary care physician after several months of worsening bilateral leg pain. He describe the pain as a deep, constant ache that worsens after walking. Vital signs are normal. Physical exam reveals an enlarged skull and bilateral hearing loss. Plain x-rays of the skull and lower extremities reveal a mixture of lytic and sclerotic lesions with thickened cortex and excessive bone formation. Laboratory studies show elevated serum alkaline phosphatase with normal calcium and phosphate levels.
Which of the following statements characterizes the initial phase of this condition?
A. increased osteoclastic activity
B. increased osteoblastic activity
C. defective mineralization of osteoid
D. reduced osteoclastic activity
E. loss of articular cartilage

A

A. increased osteoclastic activity
This patient’s physical exam, radiographic, and laboratory findings are consistent with a diagnosis of Paget’s disease of bone. There are three sequential phases of Paget disease: (1) an initial, osteolytic stage characterized by increased osteoclastic activity; (2) a mixed osteoclastic-osteoblastic stage, and (3) a sclerotic, osteoblastic stage. This condition is thought to be due to abnormalities of osteoclasts that lead to excessive bone resorption and accelerated bone formation. The resulting bone is disorganized, weak, and prone to fracture. Frequently affected bones include the skull, spine, and long bones of the lower extremities. The most common symptom is pain, but most patients are asymptomatic. Classic signs and symptoms include increased hat size and hearing loss secondary to narrowing of the auditory foramen. The condition is often diagnosed incidentally when elevated serum alkaline phosphatase levels are found on routine testing or characteristic changes are observed on x-rays obtained for other reasons.

B. Increased osteoblastic activity

              Increased osteoblastic activity is seen in the third, sclerotic phase of Paget disease.

              C. Defective mineralization of osteoid

              Defective mineralization of osteoid is seen in osteomalacia and rickets, which can occur secondary to vitamin D deficiency. X-ray of children with rickets classically shows outward bowing of the femur and/or tibia.

              D.  Reduced osteoclastic activity

              Reduced osteoclastic activity is seen in osteopetrosis (marble bone disease), in which defective osteoclasts fail to resorb bone. Patients present with frequent fractures as well as pancytopenia and extramedullary hematopoiesis due to occupation of the marrow space by bone.

              E. Loss of articular cartilage

              Loss of articular cartilage is a central process of osteoarthritis. In this condition a variety of factors, including excessive joint loading, stimulates pro-inflammatory factors and proteases that degrade extracellular matrix and destroy joint tissues including cartilage. Osteoarthritis is a slow progressive disease that usually affects individuals >40 years of age. It presents with joint pain without significant swelling and stiffness that is typically worse in the evenings. It commonly affects the cervical and lumbar spine, first carpometacarpal joint, proximal and distal interphalangeal joints, hip, knee, subtalar joint, and first metatarsophalangeal joint. Physical exam may reveal crepitus, bony enlargement, tenderness, and decreased range of movement.
53
Q

A 36-year-old man comes to the emergency department complaining of bloody diarrhea for the past 2 weeks. In addition, he reports the recent onset of a sore throat and pain in his joints. He has been intermittently febrile with a temperature reaching 39.44°C(103.0°F). On physical examination, the patient appears acutely ill, periodically complains of cramps, and is diffusely tender on palpation of the abdomen. In addition, he has multiple large, erythematous nodules on his shins that are painful to touch. Rectal exam reveals frank blood and mucus. He recalls indulging in large amounts of pork and other meats with his wife at a Brazilian steakhouse for their 15th wedding anniversary dinner 3 weeks prior to presentation.
What organism is likely responsible for this patient’s symptoms?
A. Campylobacter jejuni
B. Streptococci pyogenes
C. Yersinia enterocolitica
D. Vibrio cholerae
E. Norovirus

A

C. Yersinia enterocolitica
Given his symptoms, the patient most likely has yersiniosis from Yersinia enterocolitica. Yersiniosis often presents with enterocolitis, with a longer duration of bloody diarrhea, and painful mesenteric adenitis from the ingestion of raw or undercooked meats, milk, and untreated water. Pharyngitis can also appear as a presenting symptom in approximately 20% of patients. Adult patients can additionally develop reactive arthritis and erythema nodosum several weeks after the onset of acute infection. The symptoms of yersiniosis can mimic appendicitis or Crohn’s disease.

A. Campylobacter jejuni

              C. jejuni causes inflammatory diarrhea and is transmitted by ingestion of raw chicken and milk. In addition, C. jejuni infection commonly precedes Guillain-Barré Syndrome. However, given the prolonged duration of the diarrhea and pharyngitis seen in this patient, this is not the best answer.

              B. Streptococci pyogenes

              While erythema nodosum and pharyngitis could be caused from S. pyogenes, the organism would be unlikely to cause the gastrointestinal symptoms presented here.

              D. Vibrio cholerae

              V. cholerae usually presents with a watery “rice-water” diarrhea and is not associated with fever or bloody diarrhea.

              E. Norovirus

              Norovirus causes a self-limiting non-bloody diarrhea and is not associated with pharyngitis, bloody diarrhea, or any of the rheumatologic symptoms presented here. It is responsible for epidemics of viral gastroenteritis.
54
Q
A 43-year-old female is brought to the emergency room after experiencing worsening confusion at work. Her co-workers had suspected a stroke. The patient has no significant medical history and is on no medications. She has no allergies. Vital signs show temperature of 38.8°C (102°F), heart rate of 106, blood pressure 110/60, respirations 18, and an oxygen saturation of 97% on room air. Labs are notable for a hematocrit of 32%, platelets of 40,000/mm3, and a creatinine of 3 mg/dL.
Most likely diagnosis?
A. Hemolytic Uremic Syndrome
B. HSP
C. Idiopathic thrombocytopenic purpura
D. IgA nephropathy
E. Thrombotic thrombocytopenic purpura
A

The patient in this vignette most likely has thrombotic thrombocytopenic purpura (TTP). TTP is a rare, life-threatening emergency characterized by a classical pentad of symptoms:

1) Microangiopathic hemolytic anemia → produces schistocytes
2) Thrombocytopenia
3) Renal failure (generally look for an ↑ BUN:Cr ratio)
4) CNS changes (i.e. mental status)TTP is due to a deficiency in the vWF multimer cleaving protease ADAMTS-13. This deficiency can be acquired (e.g. auto-antibodies against ADAMTS-13). The deficient/defective protease results in larger multimers of vWF, which have a higher propensity for aggregating and forming white microthrombi in the vasculature. The white microthrombi in microvasculature can lead to organ dysfunction, notably neurologic (stroke-like sx or altered mental status) and renal failure (hematuria and ↑ BUN:Cr). The thrombi also shear RBCs, causing a microangiopathic hemolytic anemia (schistocytes are often present but not pathognomonic).

55
Q

A 2-week-old infant presents to the emergency department with fevers and poor feeding. During the past twelve hours the child has been less active than usual, has breast feed only once, and has had minimal urine output. Her temperature is 40°C (104°F), heart rate is 110 beats per minute, blood pressure is 90/48 mmHg, respiratory rate is 36 breaths per minute, and oxygen saturation is 97% on room air. On physical exam, she appears lethargic. The anterior fontanel is bulging. There is no heart murmur. Lungs are clear to auscultation bilaterally. Abdomen is soft, non-tender, and non-distended with no hepatosplenomegaly. Femoral pulses are 2+ and symmetric and capillary refill is sluggish. There is no rash. A complete blood count, blood cultures, urine cultures, and cerebral spinal fluid (CSF) studies are obtained. The CSF appears cloudy. There are 4 red blood cells per mL, 1,480 white blood cells per mL with 85% neutrophils, glucose of 35 mg/dL, and protein of 65 mg/dL. The gram stain shows gram-negative rods.

Most likely pathogen?
A. Listeria monocytogenes
B. E. coli
C. Strep agalctiae
D. Strep pneumoniae
E. Neisseria meningitidis
A

The symptoms of meningitis in young infants can be minimal and they often present with fever and poor feeding. Other signs that are present in this infant include lethargy and a bulging fontanel. Even well appearing infants in this age category require a full sepsis work-up for a fever greater than 39°C (102.2°F) since the risk of an occult bacterial infection is so high. The cerebral spinal fluid studies in this patient are consistent with bacterial meningitis: white blood cells are elevated (normal 1 month of age through adolescence, despite vaccination, as not as serotypes are contained in the childhood vaccine.

              E. Neisseria Meningitidis

              Neisseria meningitidis is a gram-negative diplococci that is more common in older children and adolescents.
56
Q

A 68-year-old male presents to the emergency department complaining of shortness of breath, dizziness, and the sensation of his heart “racing.” He says that his symptoms started abruptly earlier that day and have steadily become worse. He reports a history of long-standing hypertension, coronary artery disease with recent percutaneous transluminal angioplasty and placement of 2 stents, and paroxysmal atrial fibrillation. On admission, his heart rate is 160 beats/minute, blood pressure 100/50 mmHg, respiratory rate 26 breaths/minute, and oxygen saturation 98% on room air. A 12-lead ECG shows a narrow QRS complex tachycardia with an irregularly irregular rhythm. A diagnosis of atrial fibrillation with rapid ventricular response is made. The patient is given increasing doses of IV metoprolol without improvement and so digoxin is given as a second agent. Digoxin produces an effective response in 50% of individuals at 0.8 ng/mL, but it also produces toxic effects in 50% of individuals at 2 ng/mL. It has a half-life of 36 hours.
What is the therapeutic index of digoxin?

A

Digoxin is a cardiac glycoside often used in the treatment of various heart conditions, especially atrial fibrillation. Digoxin slows down the conduction in the AV node and increases its refractory period, thereby reducing the ventricular rate. The therapeutic index (TI) of digoxin depends on its TD50 (concentration at which 50% of individuals experience toxic effects) and ED50 (concentration at which the drug produces an efficacious response in 50% of individuals). The equation is: TI=TD50ED50\displaystyle TI = \frac {TD_{50}} {ED_{50}}. Therefore, TI=2ngmL0.8ngmL=2.5\displaystyle TI = \frac {2 \frac {ng} {mL}} {0.8 \frac {ng} {mL}} = 2.5.

57
Q

A 34-year-old woman develops the worst headache of her life and is brought to emergency department for evaluation. She is found to have subarachnoid hemorrhage secondary to a ruptured intracranial aneurysm. Her mother and both of her siblings have suffered from ruptured intracranial aneurysms.
This heritable syndrome is most likely linked to a mutation on which of the following chromosomes?
A. 13
B. 14
C. 15
D. 16
E. 17

A

The patient suffers from a ruptured intracranial aneurysm in the setting of likely autosomal dominant polycystic kidney disease (ADPKD). A gene defect in PKD1 gene encoding for the polycystin 1 protein is thought to be responsible in the majority of cases. The inheritance pattern in the vignette is consistent with the autosomal dominant inheritance pattern, given that all siblings are affected. Patients with ADPKD have a higher incidence of intracranial aneurysm formation.